Wound Healing/Keloids 01-22 Flashcards

1
Q

A 7-year-old girl presents with right facial paralysis caused by perinatal trauma. Facial reanimation is planned using a free gracilis muscle flap. When compared with the masseteric branch as the donor nerve, using a cross-facial nerve graft is more likely to provide which of the following benefits to this patient?

A) Fewer operative procedures
B) Greater commissure excursion when smiling
C) Higher axon count
D) More durable result as the patient grows
E) More spontaneous, coordinated smile

A

The correct response is Option E.

Cross-facial nerve grafts result in a more spontaneous, coordinated smile when compared with using the masseteric nerve as the donor, as it employs the patient’s natural neuronal pathway for smiling. In addition, the contralateral facial nerve that controls the commissure on the intact side also controls the commissure on the reconstructed side, the presumed reason for the more coordinated smile.

Free gracilis muscle transfer is the most frequently discussed dynamic reconstruction of elevation of the commissure (smiling) in the literature. Motor nerves used to power the gracilis include cross-facial nerve grafting, the motor nerve to the masseter muscle, the hypoglossal nerve, the spinal accessory nerve, or combinations of these motors.

Cross-facial nerve grafting requires a separate procedure to place a nerve graft from the contralateral facial nerve, adding to the overall number of procedures to achieve smiling, compared with using the masseteric nerve as donor. The number of required revision procedures is thought to be the same for both approaches. Excursion (the amount of rise of the commissure) is better with the masseteric motor nerve. Children do not seem to “outgrow” facial nerve reanimation surgeries in long term follow-up. The masseteric nerve has a higher axon count than the distal end of the cross-facial nerve graft.

How well did you know this?
1
Not at all
2
3
4
5
Perfectly
2
Q

A 7-year-old child presents to the clinic for postoperative examination after undergoing excision of a melanocytic nevus from the left thigh 9 days ago. During the past several days, the skin has become increasingly red and itchy despite the application of antibiotic ointment three times daily. Prior treatment with similar ointment did not elicit the same response. On examination, the area around the incision shows well-demarcated erythema and scaly skin. This disease process is consistent with which of the following hypersensitivity reactions?

A) Type I
B) Type II
C) Type III
D) Type IV

A

The correct response is Option D.

There are two forms of contact dermatitis: irritant and allergic. Irritant contact dermatitis is caused by the non–immune-modulated irritation of the skin by a substance, leading to skin changes. Allergic contact dermatitis is a delayed hypersensitivity reaction in which a foreign substance comes into contact with the skin; skin changes occur after reexposure to the substance. Common causes include poison ivy, nickel, and fragrances. Because this patient was exposed to antibiotic ointment with no prior response, this is likely an allergic reaction to antibiotic ointment.

Allergic contact dermatitis is a type IV, T-cell mediated, delayed hypersensitivity reaction in which a foreign substance comes into contact with the skin and is linked to skin protein, forming an antigen complex that leads to sensitization. Upon reexposure of the epidermis to the antigen, the sensitized T cells initiate an inflammatory cascade, causing the skin changes associated with allergic contact dermatitis.

There are four types of hypersensitivity reactions. Type I hypersensitivity reactions (e.g., anaphylaxis) are IgE-mediated and occur very quickly after exposure. It is associated with allergens such as bee stings, peanuts, and certain medications. Type II hypersensitivity reactions are cytotoxic/antibody-mediated reactions (e.g., hemolytic reactions, Goodpasture syndrome, and hyperacute graft rejection). Type III hypersensitivity reactions are mediated by immune complex IgG and IgM antibodies, and include certain diagnoses like hypersensitivity pneumonitis, systemic lupus erythematosus, polyarteritis nodosa, and serum sickness.

How well did you know this?
1
Not at all
2
3
4
5
Perfectly
3
Q

A 16-year-old boy presents with contracture of the proximal interphalangeal (PIP) joint after secondary healing of a volar burn of the right hand. Which of the following types of cell is responsible for contraction of this scar?

A) Keratinocyte
B) Langerhans
C) Melanocyte
D) Merkel
E) Myofibroblast

A

The correct response is Option E.

Secondary wound healing is often aided by contraction. In some cases, this contraction may be excessive, causing cosmetic and/or functional problems such as a joint contracture. Fibroblasts and myofibroblasts are responsible for this contraction. The myofibroblast is a terminally differentiated fibroblast that functions as a smooth muscle-like cell expressing alpha-smooth muscle actin. It decreases its cell length, contracting the extracellular matrix to which it is attached. These cells are found in the dermis and cause ongoing scar contraction.

Myofibroblasts produce more collagen and less collagenase and are found more frequently in hypertrophic scars versus normal human scars. They are also resistant to apoptotic signaling, which may contribute to their preponderance on hypertrophic scars.

The other listed cell types are found in normal skin. Keratinocytes produce keratin. Langerhans cells are dendritic cells specializing in antigen presentation. Melanocytes produce melanin. Merkel cells are involved in mechanoreception.

How well did you know this?
1
Not at all
2
3
4
5
Perfectly
4
Q

Which of the following is a CONTRAINDICATION to the use of finasteride as a treatment for premenopausal women with hair loss?

A) Amenorrhea
B) Galactorrhea
C) Hirsutism
D) Masculinization of facial features
E) Pregnancy

A

The correct response is Option E.

Although FDA-approved for male-pattern hair loss, finasteride is not approved by the FDA for use in women. It is classified as Pregnancy Category X (highest risk) and should not be taken or handled by pregnant women, women who may become pregnant, or those who are breast-feeding. Finasteride has been linked to abnormalities of the external genitalia of a male fetus of a pregnant woman who receives finasteride.

The other choices are not common problems with finasteride, although breast tenderness has been reported.

European studies have selectively used the drug in women for cases of hair loss associated with hyperandrogenism.

How well did you know this?
1
Not at all
2
3
4
5
Perfectly
5
Q

A 65-year-old man with C5 quadriplegia presents with a pressure ulcer of the right ischium. The ulcer has been present for several months but has been exacerbated recently since he moved into a nursing home several weeks ago. Examination of the wound shows a 5 × 5-cm open wound that extends into the subcutaneous tissue. Palpation of the base of the wound demonstrates a soft yellow eschar. There is no purulence present. The patient does not desire reconstruction, and treatment with leptospermum honey is initiated. Regarding topical wound care, which of the following is the mechanism of medical honey?

A) Absorbs wound fluid through ion exchange
B) Cleaves necrotic tissue along denatured collagen strands
C) Creates a germicidal, bacteriostatic wound environment
D) Denatures nonviable protein in the wound bed
E) Draws fluid from deep tissues to the wound surface through osmosis

A

he correct response is Option E.

For patients who are not candidates for surgery, medical management of complicated wounds is a mainstay of treatment. In addition to typical behavioral modifications (pressure-offloading, nutrition optimization, managing medical comorbidities, etc), topical wound care treatments may offer additional help in controlling the wound bed of these complicated injuries.

Honey works through two key mechanisms of action: 1) it acts as an osmotic engine to draw fluid from deeper tissues to the wound surface to promote removal of devitalized tissue; 2) it makes the wound environment more acidic and more favorable for healing.

Alginate dressings work by absorbing wound fluid through ion exchange. They come in forms including hydrogels, films, foams, and sponges, absorbing wound fluid and resulting in gels that maintain a physiologically moist environment and minimize bacterial infections.

Collagenase debrides by cleaving necrotic tissue at seven specific sites along the denatured collagen strand. The denatured collagen process creates bioactive peptide byproducts, inducing a cellular response fibroblast, keratinocyte and endothelial cell migration to the wound bed, and are associated with the proliferative phase of wound healing.

When combined together, papain and urea (Accuzyme, Panafil) bring about two supplemental chemical actions: 1) to expose by solvent action the activators of papain, and 2) to denature the nonviable protein matter in lesions, and thereby render it more susceptible to enzymatic digestion. It is no longer available in the United States.

Dakin’s solution consists of a dilute solution of sodium hypochlorite (NaClO), commonly known as bleach. The main active agent in the Dakin solution is created when the chlorine in the solution reacts with water in the environment to form hypochlorous acid (HClO). This hypochlorous acid produces the potent antibacterial effect in tissues. Unlike stronger germicidal solutions that contain carbolic acid or iodine, Dakin does not damage living cells or lose potency in the presence of blood serum. It has a solvent action on dead cells that hastens the separation of dead tissue from living tissue.

How well did you know this?
1
Not at all
2
3
4
5
Perfectly
6
Q

A 7-year-old boy of Japanese descent is brought to the clinic by his mother because she is concerned that he is “allergic to the sun.” Medical history includes sunburns, with minimal sun exposure causing severe sunburns, and significant freckling at 2 years of age. The patient’s other siblings are unaffected despite having similar complexions. A biopsy of a raised skin lesion on the cheek is performed, and results demonstrate squamous cell carcinoma. Which of the following is the inheritance pattern of this child’s most likely condition?

A) Autosomal dominant
B) Autosomal recessive
C) Multifactorial
D) X-linked dominant
E) X-linked recessive

A

The correct response is Option B.

This young boy has xeroderma pigmentosum (XP), an autosomal recessive disorder that affects approximately 1 in 250,000 patients. A defect in DNA cellular repair mechanisms makes them highly susceptible to DNA mutations leading to cutaneous malignancies at an early age. XP is present in all races, but it is more common in persons of Japanese ancestry. It is characterized by sensitivity to sunlight leading to severe sunburns, early freckle development, and early skin cancer development. XP patients younger than the age of 20 years have a 20,000-fold increased risk for nonmelanoma skin cancer, a 2000-fold increased risk for melanoma, a 100,000-fold increase in tongue cancers, and a 50-fold increase in neurologic cancers. The average age of diagnosis of a first nonmelanoma skin cancer in XP patients is 8 to 9 years old.

How well did you know this?
1
Not at all
2
3
4
5
Perfectly
7
Q

A 56-year-old man presents with a chronic wound of the plantar surface of the foot. Medical history includes type 2 diabetes mellitus. Which of the following characteristics most strongly indicates hyperbaric oxygen therapy may be beneficial for this patient?

A) Ankle brachial index greater than 1
B) Exposed bone
C) Failed total contact casting
D) Hemoglobin A1c greater than 7.0%
E) Peripheral neuropathy

A

The correct response is Option B.

Hyperbaric oxygen therapy (HBOT) is a useful adjunctive treatment for selected diabetic foot ulcers (DFUs). Studies support its use in deep ulcers to improve healing and decrease amputation risk. These include Wagner grade 3 ulcers (bone involvement, deep abscess) and grade 4 ulcers (gangrene of a portion of the forefoot). Data have not supported routine use of HBOT in DFUs involving only soft tissue (Wagner grades 1 and 2).

Serum hemoglobin A1c, which provides a measure of blood glucose over the previous 2 to 3 months, is useful for monitoring glycemic control in diabetic patients.

Peripheral neuropathy is a common finding in diabetes mellitus, and it contributes to development and persistence of chronic foot ulcers.

An ankle brachial index greater than 1 can be seen in diabetic patients but is not an indication for HBOT.

Total contact casting is a useful technique which allows off-loading of pressure so that a chronic DFU might heal. However, HBOT is only indicated in DFUs which are Wagner grade 3 or greater.

How well did you know this?
1
Not at all
2
3
4
5
Perfectly
8
Q

A 24-year-old woman who works as a radiology technologist presents to the emergency room 1 hour after her right hand was exposed to an estimated 5 Gy of radiation. Examination shows no abnormalities. Which of the following is the most likely clinical outcome?

A) Delayed dermal necrosis
B) Extensive blistering of the entire hand
C) Increased susceptibility to soft-tissue infections
D) Transient erythema, pruritus, and hair loss
E) No sequelae

A

The correct response is Option D.

The upper extremity is commonly affected in workplace and industrial fluoroscopy accidents.

Mild, or low, energy exposures (less than 10 Gy) are generally associated with a transient erythema, itching, and loss of hair. Moderate exposures (10 to 20 Gy) can have more immediate erythema that will resolve but then reoccur 1 to 2 weeks following injury. Higher exposures (greater than 20 Gy) are associated with more immediate symptoms of erythema and pain and can lead to complete tissue necrosis. Pseudomonas infection has been reported in these injuries but is more commonly involved in more severe cases that necessitate amputation.

How well did you know this?
1
Not at all
2
3
4
5
Perfectly
9
Q

A 45-year-old man undergoes excision of a sarcoma from the leg. The resulting wound is 6 x 5 cm in size. Wound closure is not performed so that the margin’s status can be assessed with final pathology prior to definitive reconstruction. It is thought that the wound will have heavy drainage because of edema of the limb. Therefore, a dressing with good capacity to absorb exudate is desired. Which of the following is the most appropriate management for this wound?

A) Foam dressing
B) Hydrocolloid dressing
C) Hydrofiber dressing
D) Hydrogel dressing

A

The correct response is Option C.

Many different kinds of wound care products are available, each with their own characteristics. The choice of dressing is dependent on the type of wound, product availability, and cost. Hydrofibers are highly absorbent dressings that are composed of hydroxymethyl cellulose fibers. On contact with water, they turn into a gel and thus maintain a moist wound environment. They can be left on the wound for several days, resulting in less frequent need for dressing changes. They are typically used for wounds with moderate-to-high exudate.

Hydrogels are cross-linked hydrophilic polymers with a high (90%) water content. They are available in the form of gels and sheets. They create a moist wound environment. Their highly aqueous nature makes dressing changes atraumatic. They do not absorb exudate. They are used for dry and minimally exudative wounds, like eschars and granulation tissue.

Hydrocolloids consist of cross-linked carboxymethyl cellulose, gelatin, and pectin. They absorb wound exudate and form a gel. However, their absorptive capacity is much less than that of hydrofibers. They are used for mild-to-moderately exudative wounds.

Foams are semi-occlusive dressings that have a porous open-cell architecture. They are usually composed of two layers: an inner hydrophilic absorbent contact layer and an outer hydrophobic protective layer. They maintain a moist wound environment and protect the wound from trauma. Their degree of absorption depends on the thickness of the foam layer. They are used for mild-to-moderately exudative wounds.

How well did you know this?
1
Not at all
2
3
4
5
Perfectly
10
Q

A 75-year-old man with dilated cardiomyopathy is seen in the intensive care unit (ICU). He was initially admitted for sepsis 6 weeks prior and has remained in the ICU throughout his course. The primary team noticed anorexia, weakness, a diffuse nonpruritic perifollicular rash, and poor wound healing of minor scrapes and bruises. Ascorbic acid (vitamin C) level is 5 ?mol/L (N 45 to 90 ?mol/L). This ascorbic acid level is most likely to impair the properties of lysyl oxidase and which of the following events in the wound-healing cascade?

A) Collagen cross-linking
B) Collagen degradation
C) Collagen protein translation
D) Procollagen cleavage
E) Procollagen trafficking

A

The correct response is Option A.

Wound healing is a dynamic process and is dependent on numerous enzymes to catalyze the formation of new collagen fibrils. Lysyl oxidase is a particularly important enzyme in the wound healing cascade and is required for collagen cross-linking. Procollagen C- and N-proteinases cleave carboxyl- and amino-terminal propeptides of procollagens. It remains unclear which enzymes are involved in procollagen trafficking, but this action is thought to be mediated by COP-I and COP-II transport complexes. Collagen protein translation proceeds via standard ribosomes and degradation is mediated by collagenase.

How well did you know this?
1
Not at all
2
3
4
5
Perfectly
11
Q

A 42-year-old woman is scheduled to undergo autologous breast reconstruction. Which of the following is the most likely effect of steroid use in this patient?

A) Long-term corticosteroid use is associated with increased risk of free flap failure
B) Single perioperative corticosteroid dose is associated with transient hyperglycemia
C) Single perioperative corticosteroid dose negatively affects wound healing
D) The use of vitamin E counteracts the negative effects of corticosteroids on wound healing

A

The correct response is Option B.

The effects of corticosteroids on wound healing have been extensively studied. A single perioperative dose has not been associated with wound healing problems or complications. There is, however, a mild increase in glycemia, even in patients without diabetes.

The long-term use of corticosteroids has been associated with increased wound complications in susceptible individuals. It depends on the dose and duration of corticosteroid treatment.

The use of vitamin A, not E, has been shown to counteract the negative effects of corticosteroids on wound healing.

How well did you know this?
1
Not at all
2
3
4
5
Perfectly
12
Q

Patients who receive immunosuppressive therapy after undergoing allograft transplantation are most likely to develop infections from opportunistic pathogens during which of the following postoperative (posttransplantation) periods?

A) After the first month and within 1 year
B) After the first week and within 1 month
C) After the first year
D) Between days 3 and 7
E) Within 48 hours

A

The correct response is Option A.

Patients receiving immunosuppressive therapy after allograft transplantation are most likely to develop infections caused by opportunistic pathogens after the first month and within 1 year after transplantation.

Classically, infections in immunosuppressed patients after organ transplantation occur in three distinct time periods. 1) During the first month after transplantation, most infections are typical postoperative nosocomial infections, although donor-derived and pre-existing recipient infections can also occur. 2) During the period of peak immunosuppression, typically between 1 and 12 months after transplantation, most infections are classic opportunistic infections, including cytomegalovirus, herpes simplex virus, herpes zoster, Epstein-Barr virus, Aspergillus, Nocardia, and Pneumocystis. Use of prophylaxis may result in a later onset of opportunistic infections. 3) After 12 months posttransplantation, recipients with satisfactory allograft function will tolerate decreased maintenance immunosuppression, with a lowered risk for developing infections by classic opportunistic agents, although they may still occur. Community-acquired infections are most typical during this period, as healthy recipients suffer community-based epidemiological exposures. These infections may be more prolonged and result in more complications than in otherwise healthy patients.

How well did you know this?
1
Not at all
2
3
4
5
Perfectly
13
Q

A 29-year-old African American man presents with recurrence of an earlobe keloid 16 months after excision and closure. Re-excision followed by hypofractionated electron-beam radiation therapy is planned. Regarding timing of the initiation of radiation, which of the following is associated with the lowest rate of keloid recurrence?

A) Less than 24 hours after excision
B) 24 to 48 hours after excision
C) 2 to 4 days after excision
D) 7 to 10 days after excision
E) 30 to 60 days after excision

A

The correct response is Option A.

The use of hypofractionated electron-beam radiation therapy has been shown to reduce the probability of keloid recurrence after excision. A retrospective study by Shen et al. of 834 keloids treated with 18 Gy in two fractions a week apart showed that intervals of 24 to 48 hours between surgery and radiotherapy (hazard ratio = 1.52, p less than 0.02) and of greater than 48 hours (hazard ratio = 1.77, p less than 0.02) showed an increased relapse rate compared with the interval of less than or equal to 24 hours. Other predictors of relapse included male gender, age less than or equal to 29 years old, keloids greater than 5 cm, keloids in high-tension areas, and keloids that required skin grafts for reconstruction.

How well did you know this?
1
Not at all
2
3
4
5
Perfectly
14
Q

A 45-year-old African American woman who underwent cardiac surgery 6 months ago seeks cosmesis of the surgical scar. Physical examination shows a raised, thick keloid scar that is pruritic. Which of the following nonsurgical treatments is likely to produce the most rapid improvement in the scar with the fewest adverse effects?

A) Intralesional 5-fluorouracil
B) Intralesional triamcinolone
C) Radiation therapy
D) Silicone sheeting
E) Topical triamcinolone

A

The correct response is Option A.

Injection of 5-fluorouracil demonstrates similar efficacy to intralesional corticosteroid therapy (triamcinolone) and has the advantage of a lower risk of hypopigmentation.

Silicone sheeting is effective for hypertrophic scars but requires serial application and is slower in visible effect. Topical corticosteroids are not as effective as intralesional treatment. Radiation carries the risk of adjacent tissue toxicity and also includes the risk of skin pigmentation changes.

How well did you know this?
1
Not at all
2
3
4
5
Perfectly
15
Q

A 23-year-old African-American man presents with a raised thickened scar on his anterior chest that he complains is pruritic and unattractive. It was removed by another provider 4 years earlier and has slowly recurred over the past year. On examination, the lesion extends beyond the initial borders of the scar and is firm and hyper-pigmented. On review of his prior pathology report, which of the following histologic characteristics is most likely?

A) Greater ratio of type III to type I collagen
B) Multitude of myofibroblasts and smooth muscle actin
C) Parallel collagen bundles
D) Thick, wavy, and randomly oriented collagen fibers

A

The correct response is Option D.

In patients with abnormal or excessive scar tissue formation, treatment and prognosis will be driven by the correct diagnosis of a keloid versus a hypertrophic scar. This patient presents with a recurrent keloid of the chest. His clinical history supports this diagnosis by recurrence after resection, growth extending beyond the original border of the lesion, late recurrence after several years, and continued growth over several years without regression or improvement. Hypertrophic scars are less likely to recur, contained within the original boundaries of the lesion, often regress somewhat within a year, and recur earlier in the postoperative period if they are to recur. Both hypertrophic scars and keloid scars can be pruritic.

Pathologic analysis of keloids reveals more type I collagen than type III collagen, similar to normal skin. Hypertrophic scars will exhibit increased type III collagen and pro-fibrotic collagen cross-linking. Keloid growth is thought to be impacted by cell-signaling between keratinocytes and fibroblasts, but hypertrophic scar production requires an abundance of myofibroblasts expressing smooth muscle actin. While hypertrophic scars have parallel collagen fibrils and bundles, keloids are characterized histologically by thick, randomly oriented collagen fibrils that are not organized into bundles.

How well did you know this?
1
Not at all
2
3
4
5
Perfectly
16
Q

A 7-year-old boy presents with a chief complaint of multiple wide and thin scars from skin lesion excisions. The patient’s mother reports a history of late walking, hypermobile joints, and easy bruising. On the basis of these complaints, which of the following is the most likely diagnosis?

A) Capillary fragility syndrome
B) Cutis laxa
C) Ehlers-Danlos syndrome
D) Marfan syndrome
E) Osteogenesis imperfecta

A

The correct response is Option C.

Ehlers-Danlos syndrome (EDS) is the only diagnosis listed which encompasses all of the mentioned patient complaints. EDS encompasses a range of connective tissue disorders that share joint hypermobility, skin hyperextensibility, and tissue fragility. The different types of EDS have varying modes of inheritance and a variety of identified genetic mutations in collagen formation. Classical EDS is caused by an autosomal dominant mutation involving the COL5A1 or COL5A2 genes, which are involved in collagen type V synthesis. Common symptoms include skin hyperextensibility, widened atrophic “cigarette paper” scars, generalized joint hypermobility involving three or more joints, and easy bruising. Vascular EDS carries major risks, including arterial (aortic) dissection, rupture, or aneurysm. Cutis laxa is not associated with easy bruising. Capillary fragility syndrome typically does not present with late walking. Osteogenesis imperfecta and Marfan Syndrome are not associated with atrophic scarring.

How well did you know this?
1
Not at all
2
3
4
5
Perfectly
17
Q

A 63-year-old man undergoes microdermabrasion for scar irregularity following treatment of facial skin cancer. Which of the following intraoperative findings signals the endpoint of treatment?

A) Cobblestoned, yellow adiposity
B) Confluent patches of bleeding lakes on a yellowish background
C) Minimal dermis with visible subdermal plexus
D) Pinpoint, punctate bleeding on a white background
E) Thinned but present epidermis

A

The correct response is Option D.

When performing microdermabrasion, the most appropriate endpoint is removal of soft tissue into the papillary dermis. The appropriate level in the papillary dermis is visualized as pinpoint bleeding in a white dermal background. Epithelial removal would be insufficient. Removal of tissue into the reticular dermis, either superficial or deep, increases the risk for scarring. The superficial reticular dermis demonstrates a yellowish white coloration and would reinforce the need to discontinue treatment before deeper injury occurs. Furthermore, dermal excisional depth would manifest as confluent red bleeding in a yellowish background of dermis or nearly complete dermal removal with a visible subdermal vascular plexus. If a full-thickness skin removal is performed (into the subcutaneous adipose layer), significant deformity could result. Preservation of adnexal structures to allow reepithelialization is a key tenet in determining the appropriate depth of treatment. Posttreatment changes include thickened epidermis and increased elastin and collagen.

How well did you know this?
1
Not at all
2
3
4
5
Perfectly
18
Q

An 80-year-old woman undergoes excision and direct closure of a nasal sidewall carcinoma. The wound is closed with cuticular nylon suture and dressed with a temporary sterile gauze bandage in an uncomplicated office procedure with immediate discharge home. How long after surgery should this patient be advised to refrain from wetting the suture line with tap water rinses?

A) 48 Hours after surgery
B) 48 Hours after suture removal
C) Until suture removal
D) Until wound edge epithelization is complete
E) No restriction

A

The correct response is Option E.

Several studies have compared wet, moist, and dry wound healing following skin surgery without demonstrating an increase in infection rate when washing the wound with tap water at any point postoperatively as opposed to keeping the site dry for various lengths of time. This includes a rigorous randomized control trial in which patients with defects following skin lesion removal were divided into groups with tap water wound washing within 12 hours of surgery versus those asked to keep wound dry for 48 hours, where the incidence of surgical site infection in the wash group was not inferior to the dry group. On the other hand, unrestricted wound washing improves patient comfort, and multiple studies have demonstrated that wet or moist wounds promote reepithelialization and result in reduced scar formation with less inflammatory reaction compared to dry wounds.

How well did you know this?
1
Not at all
2
3
4
5
Perfectly
19
Q

Which of the following is the primary role of adipose-derived stem cells (ADSC) in wound healing?

A) Assist in chemotaxis of platelets and granulocytes
B) Differentiate directly into fibroblasts and keratinocytes
C) Induce development of hair and sweat follicles
D) Provide a scaffold for deposition of granulation tissues
E) Register and organize pro-collagen fibrils

A

The correct response is Option B.

Adipose-derived stems cells (ADSC) have had extensive study in vitro and in vivo because there are ready sources of them from adult patients, which bypasses many ethical and regulatory issues of embryonic stem cells.

ADSC have both direct structural and paracrine roles in wound healing. They can directly differentiate into keratinocytes, endothelial cells, and dermal fibroblasts. ADSCs, through paracrine phenomena, are modulators of the inflammatory environment of the wound healing milieu but are not involved in the immediate chemotaxis during the inflammatory period nor do they function as a scaffold during the proliferative phase. Lysyl oxidase is the extracellular enzyme responsible for final alignment of collagen fibrils.

Presence of skin adnexa such as hair follicles and sweat glands are hallmarks of scarless, fetal healing. Hair follicle formation typically only occurs during embryonic development and involves interaction of ectodermal and mesenchymal cells influenced by signaling pathways including Wnt/b-catenin and BMPl but not ADSCs.

How well did you know this?
1
Not at all
2
3
4
5
Perfectly
20
Q

Which of these situations is best suited for the use of a topical skin adhesive (polymerizing cyanoacrylate) for closure?

A) Burst laceration along the eyebrow
B) Over an intradermal repair of a vertical forehead laceration
C) Over a suture repaired dog bite of the ear
D) Straight line laceration on the cheek with 4 mm of separation
E) Well apposed lip laceration crossing the vermilion cutaneous border

A

The correct response is Option B.

Cyanoacrylate skin adhesives are sold as monomers that polymerize by an exothermic reaction on contact with air and fluids. They can be used in conjunction with other skin closure mechanisms such as sutures or as a primary skin closure device. There is evidence across multiple surgical specialties and situations that skin glues can save time in the operating room. When used correctly, the cosmesis is similar or better than external suturing. There are in vitro studies suggesting inhibition of Gram-positive cocci and clinical anecdotal evidence of decreased infection.

There is evidence that when wounds are closed with skin glue as the only closure device that dehiscence rates are increased.

If the adhesive leaks below the skin when applied, it can hold the edges open and delay or prevent healing with increased scarring or poorer cosmesis. Adhesives should be applied to well apposed skin edges only.

The U.S. Food & Drug Administration–approved package insert for a major brand of skin adhesive (eg, Johnson & Johnson, Ethicon Dermabond) specifically indicates use on mucosa and over dirty wounds such as dog bites to be contraindicated.

A burst laceration along the eyebrow and a straight line laceration on the cheek with 4 mm of separation would be contraindicated because of the lack of excellent epithelial continuity. Adhesive over a suture-repaired dog bite of the ear and a well-opposed lip laceration crossing the vermilion cutaneous border are specifically contraindicated on the package insert. Studies across multiple surgical specialties are supportive of cyanoacrylate skin adhesives over an intradermal repair of a vertical forehead laceration.

How well did you know this?
1
Not at all
2
3
4
5
Perfectly
21
Q

A 46-year-old woman undergoes a fleur-de-lis abdominoplasty following successful gastric bypass surgery. She has a 94-lb (43-kg) weight loss and current BMI is 28 kg/m2. To ensure the best outcome for wound healing, which of the following is the recommended postoperative daily intake of protein for this patient?

A) 20 to 30 g/day
B) 40 to 50 g/day
C) 60 to 70 g/day
D) 80 to 90 g/day
E) More than 100 g/day

A

The correct response is Option C.

Nutritional status in postbariatric subjects is essential in achieving successful healing of surgical wounds. Anatomical changes to the gastrointestinal tract following bariatric surgery can exacerbate nutritional deficiencies and inadequacies. Reduced protein intake has been associated with significantly lower healing rates in massive weight loss patients. It is recommended that postbariatric patients consume 60 to 70 g/day of protein 2 to 4 weeks prior to surgery and for 1 to 2 months postoperatively.

How well did you know this?
1
Not at all
2
3
4
5
Perfectly
22
Q

A 58-year-old woman develops full-thickness dermal necrosis in a 4 × 4 × 2-cm area of her lower breast following reduction mammaplasty. After debridement to healthy tissue, she starts daily wound packing with a calcium alginate fiber dressing. The main advantage of calcium alginate versus saline gauze dressings is a decrease in which of the following?

A) Dressing change frequency
B) Healing time
C) Keloid scarring
D) Treatment cost
E) Wound infection rate

A

The correct response is Option A.

An effective dressing should aid in surface debridement, absorb wound exudate, and maintain a moist healing environment. Normal saline wet-to-dry gauze dressings have been a mainstay of wound management for generations because they are easy to perform, widely available, and inexpensive. They are best changed 2 to 3 times daily to remove exudative material because they can quickly become saturated. Some of the wound healing byproducts, such as metalloproteinases and elastase, can slow down wound healing and result in chronic wounds. By wicking away these potentially harmful agents, more absorptive wound dressings can help simplify care. Alternatives to conventional saline wet-to-dry gauze dressing materials include hydrogels, hydrocolloids, foams, alginates, and negative pressure dressings. They are more expensive than traditional saline-gauze dressings but are typically far more absorptive, allowing for less frequent dressing changes. Daily dressing changes versus two to three times a day are far more convenient for patients and may ultimately save total treatment costs by allowing for fewer nursing visits or allowing for outpatient care. Many studies show no difference in healing times, though some studies suggest a mild benefit in diabetic foot ulcers. Current recommendations call for additional studies, as evidence of faster healing times is lacking. No studies show lower infection or scarring.

Calcium alginates are fibers made of brown seaweed fibers, and they can hold more than ten times their weight in fluid. Some manufacturers claim that they are able to deactivate metalloproteinases and stimulate healing, although in vitro data are lacking. They are a comfortable and effective alternative to saline wet-to-dry dressings, albeit at a higher product cost.

How well did you know this?
1
Not at all
2
3
4
5
Perfectly
23
Q

A 30-year-old man presents with a large open wound to the right thigh. The proposed treatment plan after debridement and establishing a clean wound is to use negative pressure wound therapy (NPWT). Which of the following is the main direct mechanism for wound healing by this method?

A) Improvement in tissue auto-debridement
B) Increase in collagen synthesis
C) Reduction in wound bacterial load
D) Removal of interstitial fluid leading to increased blood flow

A

The correct response is Option D.

Based on the original studies by Moryk, it was hypothesized that the beneficial wound-healing effects of negative pressure wound therapy (NPWT) was a combination of a fluid-based mechanism and a mechanical stress mechanism. The fluid-based mechanism involves the removal of excess interstitial fluid from the wound bed by the vacuum, which results in the interstitial pressure decreasing below the capillary filling pressures, thus allowing “re-opening” of these wound bed capillaries. This leads to improved blood flow within the wound, allowing for granulation tissue formation. The mechanical strain mechanism is created by micro-strain forces created by the vacuum on the cells within the wound. Cellular deformation leads to numerous molecular changes, including activation of the vascular endothelial cell growth factor (VEGF) pathway, which enhance angiogenesis.

Collagen synthesis is not directly affected by NPWT. There is equivocal evidence for whether there is a positive or negative effect of NPWT on wound bacterial loads.

NPWT does not auto-debride wounds. It is important when using NPWT to ensure adequate mechanical debridement of nonviable tissues from the wound bed prior to initiating NPWT.

How well did you know this?
1
Not at all
2
3
4
5
Perfectly
24
Q

A 23-year-old woman comes to the office for evaluation of bilateral ear keloids. She reports that the keloids developed after she had her ears pierced 5 years ago. She has not had previous surgery for this problem. Which of the following is the most likely recurrence rate after surgical excision with injection of a corticosteroid?

A) 5%
B) 15%
C) 35%
D) 50%
E) 75%

A

The correct response is Option B.

Earlobe keloid formation after piercing is reported to affect approximately 2.5% of the population. Various adjuvant therapeutic modalities, including radiation therapy, intralesional corticosteroids, interferon, 5-fluorouracil, topical silicone, and pressure devices, are used to decrease recurrence rate after surgical excision. A recent meta-analysis looked at the recurrence rate of keloid formation after surgical excision with the use of radiation therapy and intralesional corticosteroids. Recurrence rate after excision with radiation therapy was found to be 14%. The recurrence rate after excision with intralesional corticosteroids was 15.4%. Although radiation therapy had an overall reduced recurrence rate, it was associated with higher cost and more significant potential complications. Five cases of carcinogenesis after radiation therapy have been reported. The main disadvantage of corticosteroid injections was found in most studies to be the pain of injection. Adjuvant corticosteroid injections in conjunction with surgery were performed preoperatively, intraoperatively, or postoperatively in various studies.

How well did you know this?
1
Not at all
2
3
4
5
Perfectly
25
Q

E-cigarettes are similar to traditional cigarettes in that the negative effect of nicotine on wound healing is primarily due to which of the following?

A) Endothelial cell injury
B) Increased inflammatory response
C) Increased prostacyclin production
D) Peripheral microvascular vasoconstriction
E) Platelet aggregation

A

The correct response is Option D.

Cigarette smoke contains thousands of different chemical substances including chemical toxins and carcinogens in addition to nicotine. While e-cigarettes are believed to have fewer health risks because tar and many other harmful chemicals have been removed, they do deliver vaporized nicotine. Nicotine-induced vasoconstriction in the periphery leads to relative skin ischemia and poor wound healing.

Platelet aggregation is incorrect. Nicotine does not have a direct effect on platelets. In some studies, platelet aggregation appears to be reduced with long-term administration of nicotine. Increased prostacyclin production is incorrect.

Prostacyclin is a local vasodilator with effects that minimize platelet aggregation.

Nicotine has been shown to inhibit synthesis of prostacyclin. Increased inflammatory response is incorrect. Nicotine causes a diminished inflammatory response by a weakened chemotaxis, reduced migration, impaired bacterial killing by inflammatory cells and a subnormal release of proteolytic enzymes and inhibitors. Endothelial cell injury is incorrect. While nicotine can alter the structural and functional characteristics of vascular smooth muscle and endothelial cells, it does not cause cell injury. Several studies show increased number of endothelial cells in venous blood after nicotine administration in cigarette smokers, but not in non-cigarette smokers, suggesting that other harmful chemicals found in tobacco may contribute to cell injury.

How well did you know this?
1
Not at all
2
3
4
5
Perfectly
26
Q

Hyperbaric oxygen therapy (HBOT) is most appropriate for a patient with which of the following conditions?

A) Acute osteomyelitis of the tibia
B) Anaerobic necrotizing soft-tissue infection
C) Chemical burn because of lye exposure
D) Stevens-Johnson syndrome
E) Wagner grade 2 diabetic foot ulcer

A

The correct response is Option B.

Hyperbaric oxygen therapy (HBOT) is an accepted adjunct to surgical debridement, appropriate antibiotic therapy, and indicated critical care measures for necrotizing soft-tissue infections such as necrotizing fasciitis and Fournier gangrene. The increased oxygen delivery of HBOT improves leukocyte function and can enhance penetration of certain antibiotics such as aminoglycosides. The clinical effects include slowing of the progress of the infection and decreased risk of both amputation and mortality.

There is not adequate evidence to justify HBOT in diabetic foot ulcers (DFUs) with Wagner grade 2 (extension to bone, tendon, or capsule) or less. However, there is moderate evidence to suggest benefit in DFUs with Wagner grade 3 (deep ulcer with osteomyelitis or abscess) or greater.

HBOT may be indicated in the treatment of chronic osteomyelitis but not in the acute setting. It plays no role in the treatment of Stevens Johnson syndrome, and may be beneficial in acute thermal burns but is not indicated for chemical burns.

The complete list of approved indications for HBOT, as determined by the Undersea and Hyperbaric Medical Society and the U.S. Food and Drug Administration, includes the following:

Air or gas embolism

Carbon monoxide poisoning

Clostridial myositis and myonecrosis (gas gangrene)

Crush injury, compartment syndrome, and other acute traumatic ischemias

Decompression sickness

Arterial insufficiency

Severe anemia

Intracranial abscess

Necrotizing soft-tissue infections

Refractory osteomyelitis

Delayed radiation injury (soft tissue and bony necrosis)

Compromised grafts and flaps

Acute thermal burn injury

Idiopathic sudden sensorineural hearing loss

How well did you know this?
1
Not at all
2
3
4
5
Perfectly
27
Q

Which of the following mechanisms is most likely to inhibit normal wound healing in a patient who smokes cigarettes?

A) DNA strand breaks and helical cross-linking
B) Increased cosubstrate for enzymes involved in collagen production
C) Increased platelet aggregation
D) Increased tissue oxygen delivery
E) Nicotine-induced vasodilation

A

The correct response is Option C.

The detrimental effects of smoking on wound healing are due primarily to nicotine, carbon monoxide, and hydrogen cyanide. One of the effects of nicotine is increased platelet aggregation due to enhanced adhesiveness of the platelets themselves. This leads to thrombus formation and decreased oxygen delivery. Nicotine does not produce vasodilation, but rather vasoconstriction. Both of these effects can lead to local tissue ischemia, which inhibits the normal wound healing process.

One of the major mechanisms by which ionizing radiation inhibits wound healing is production of DNA strand breaks and helical cross-linking, but smoking is not significantly involved.

Vitamin C is the vitamin which plays the greatest role in wound healing. It is required as a cosubstrate for hydroxylase enzymes, which are involved in the production of collagen. Vitamin C deficiency has long been known to inhibit wound healing (scurvy). However, supplemental vitamin C in the nondeficient patient has not been shown conclusively to produce any beneficial wound-healing effects.

How well did you know this?
1
Not at all
2
3
4
5
Perfectly
28
Q

Compared with standard wound dressings, postoperative negative pressure wound therapy is most likely to produce which of the following outcomes?

A) Better delayed primary fascial closure rates for salvage laparotomy
B) Better split-thickness skin graft incorporation
C) Increased inflammatory response
D) Increased postoperative dressing changes
E) Increased risk of infection

A

The correct response is Option B.

Multiple studies have shown the benefits of negative pressure wound therapy (NPWT) when used in conjunction with skin grafts, both as a bolster over a skin graft as well as wound bed preparation.

NPWT has been shown to decrease the risk of infection in complex and traumatic wounds in some studies, while others have shown no difference in infection rates in complex wounds when the patient has multiple comorbidities or when used to cover uncomplicated incisions for elective orthopedic operations. However, there is no evidence to suggest NPWT increases infection risk compared with standard wound dressings.

NPWT reduces both inflammatory response and edema formation.

When used for damage control laparotomy and abdominal compartment syndrome, studies have failed to show any benefit of NPWT over standard dressings. Furthermore, at least one study has suggested an increased rate of enteric fistula formation is associated with NPWT.

NPWT has been shown to decrease both the number of postoperative dressing changes and the number of additional operative interventions in complicated diabetic wounds.

How well did you know this?
1
Not at all
2
3
4
5
Perfectly
29
Q

A 48-year-old woman undergoes excision of a 3-cm recurrent keloid of the presternal chest. Immediate reconstruction with a collagen-glycosaminoglycan scaffold dermal regeneration template is performed, followed by thin (0.008-in) epidermal autografting 21 days later. After it has healed completely, punch biopsy is performed. The absence of which of the following histologic features is most likely to indicate regenerated skin in this patient?

A) Capillary loops at the dermal-epidermal junction
B) Elastic fibers
C) Hair follicles
D) Neovascularization
E) Rete ridges

A

The correct response is Option C.

Regenerated skin is clearly quite different histologically from scar and, in fact, shares many characteristics with normal physiologic skin. Regenerated skin shows mechanical competence, vascularization, and heat and cold sensitivity. Furthermore, the dermal-epidermal junction shows formation of rete ridges and capillary loops. Regenerated skin displays elastic fibers and increased collagen fiber density in the reticular dermis, and it often exhibits nerve fiber regeneration as well. Regenerated skin, even when resurfaced with a split-thickness skin graft, however, does not have the dermal appendages such as hair follicles and sweat glands, that are present throughout normal skin.

How well did you know this?
1
Not at all
2
3
4
5
Perfectly
30
Q

A 52-year-old man presents with a chronic ulcer of the lower extremity. Current medications include prednisone for management of rheumatoid arthritis. In addition to standard local wound care, which of the following treatments is most appropriate?

A) Folate
B) Hyperbaric oxygen therapy (HBOT)
C) Long-acting insulin
D) Vitamin A
E) Vitamin C

A

The correct response is Option D.

Malnutrition is a well-established risk factor for the development of chronic wounds. Vitamin A has been shown in multiple studies to offset the detrimental effects of corticosteroids on wound healing.

Appropriate glucose management is critical to the treatment of diabetic ulcers, but insulin would not be indicated in the absence of uncontrolled diabetes mellitus.

Vitamin C is required as a cosubstrate for enzymes involved in collagen production, and its supplementation is recommended for the nutritionally deficient. However, vitamin C has not been shown to be of any benefit to wound healing in the setting of chronic corticosteroid therapy per se.

Hyperbaric oxygen therapy delivers oxygen to tissues by both hemoglobin-dependent transport and vastly increased dissolved oxygen content in blood when a patient breathes 100% oxygen at pressures well above atmospheric level. This improves wound healing by multiple cellular mechanisms in select wounds. However, benefits in treatment of chronic corticosteroid use have not been demonstrated.

Elevated serum homocysteine has been associated in multiple studies with impaired wound healing and increased risk of coronary and cerebrovascular disease due to its enhancement of clotting pathways. Folate supplementation is often used to treat hyperhomocysteinemia, but conclusive benefit in chronic wounds is not as well-established.

How well did you know this?
1
Not at all
2
3
4
5
Perfectly
31
Q

A 77-year-old man undergoes wide local excision of a melanoma on the posterior shoulder. Concomitant sentinel lymph node biopsy is positive for metastasis. Completion axillary lymph node dissection and adjuvant radiation therapy to the axilla are performed. Metastatic workup shows no abnormalities. Postoperatively, the patient develops chronically draining seroma and open wound to the axilla. He is referred for management of the radiation wound after it fails to improve with conservative local wound care. Physical examination shows a 2-cm open wound to the axilla, which tunnels 5 cm into the apex and is surrounded by extensive fibrosis and erythema. Which of the following interventions is most likely to result in a definitive closure?

A) Administration of 30 hyperbaric oxygen treatments at 2.4 ATA
B) Application of negative pressure wound therapy
C) Excision of the wound cavity and full-thickness skin graft reconstruction
D) Excision of the wound edges and application of acellular dermal matrix
E) Wide excision of the irradiated soft-tissues and repair with a thoracodorsal artery perforator flap

A

The correct response is Option E.

Radiation damage produces a hypoxic, hypovascular, and hypocellular environment that can lead to delayed healing and even ulcer formation. Principles of management once a chronic radiation wound has developed include aggressive surgical removal of the entire zone of injury, which is sometimes larger than initially anticipated, and repair with well-vascularized tissue. A thoracodorsal artery perforator flap will provide sufficient healthy and supple soft tissue to cover the entire axilla. The thoracodorsal pedicle should be intact as it is not regularly divided during axillary lymph node dissection.

Negative pressure wound therapy would be contraindicated in this wound because of the potential for exposure of blood vessels in the axilla as well as for sponge retention in the tunneled wound and is, therefore, incorrect.

Excision of the wound edges only may be inadequate treatment of the zone of injury, and it is not the best choice. Furthermore, acellular dermal allograft plays no role, aside from possibly a temporizing measure, in the treatment of radiation ulcers.

Hyperbaric oxygen therapy is indicated for soft tissue radionecrosis. This delivers increased levels of oxygen to the hypoxic, irradiated wound and establishes a steeper oxygen gradient between the wound and surrounding tissues. The cellular response to this gradient stimulates neovascularization and can improve or even heal some wounds. However, with this large, cavitary wound, hyperbaric oxygen therapy, as a single-line treatment, would be unlikely to heal the wound completely.

Excision of the wound cavity and full-thickness skin graft reconstruction is not the best option for two reasons. First, the excision may lead to exposure of neurovascular structures that should not be covered with a skin graft. Second, risk of partial or total loss of a full-thickness skin graft may be unacceptably high in an irradiated wound bed.

How well did you know this?
1
Not at all
2
3
4
5
Perfectly
32
Q

During which of the following phases of wound healing is collagen deposition the greatest?

A) Collagen deposition occurs equally during all phases of wound healing
B) Collagen deposition does not occur during wound healing
C) Inflammatory
D) Proliferative
E) Remodeling

A

The correct response is Option D.

The proliferative phase of wound healing occurs in two phases. Fibrin and fibronectin are formed during the initial proliferative period. Around day 3 of wound healing, fibroblasts appear and begin production of collagen. The proliferative phase ends between 2 to 4 weeks of wound healing when collagen accumulation reaches a maximum and collagen remodeling begins, marking the beginning of the remodeling phase of wound healing.

Inflammatory phase is dominated by white blood cells. There is an influx of polymorphonuclear leukocytes (PMNs), macrophages, and lymphocytes. PMNs are not essential to wound healing, but macrophages are essential. Sterile incisions normally heal without PMNs.

Type III collagen is converted to type I in the remodeling phase. In this phase there are increased collagen cross-linking and increased tensile strength.

Collagen deposition does not occur during wound healing is incorrect. It occurs during all phases of wound healing.

Collagen deposition occurs equally during all phases of wounding healing is incorrect because collagen deposition is heaviest during the proliferative phase. Fibroblasts differentiate from resting mesenchymal cells in connective tissue. They are chemoattracted to the site, divide and produce components of ECM and their primary function is to synthesize collagen during that phase.

How well did you know this?
1
Not at all
2
3
4
5
Perfectly
33
Q

A 23-year-old woman comes to the office because of a hypertrophic scar after undergoing abdominoplasty 3 months ago. A multimodal approach to improving the appearance of the scar is planned. Which of the following therapies is supported by the highest quality evidence in this patient?

A) Allium cepa extract
B) Fat injection
C) Microneedling
D) Silicone gel
E) Vitamin E

A

The correct response is Option D.

Silicone gel has demonstrated efficacy in improving hypertrophic scars in a number of studies and is supported by level I evidence. Vitamin E, fat injection, allium cepa extract and microneedling are supported by lesser quality studies in a recent comprehensive review of the literature.

How well did you know this?
1
Not at all
2
3
4
5
Perfectly
34
Q

Use of negative pressure wound therapy with a sponge dressing is likely to result in an adverse outcome in which of the following clinical scenarios?

A) A hand wound with exposed tendons
B) A lower extremity wound with exposed femoral vessels
C) A Stage IV sacral pressure sore
D) As a bolster over a split thickness skin graft on the arm
E) Over a closed surgical incision after hernia repair

A

The correct response is Option B.

Negative pressure wound therapy (NPWT) has dramatically improved our ability to manage complicated and complex wounds. Advantages of NPWT include decreased healing time, simplified wound care with less frequent dressing changes, and promotion of healthy granulation tissue. However, there are several contraindications to the use of NPWT. The presence of exposed vessels is an absolute contraindication to its use; arterial rupture has been reported with multiple fatalities. In addition to placement over exposed vessels, significant bleeding has also been noted with use of NPWT in groin and sternal wounds, in patients on blood-thinning medications, and during removal of well-integrated foam pieces. Other complications include infections due to retained pieces of foam.

In addition to exposed vessels, NPWT should not be used in the setting of active, uncontrolled infection, malignant wounds, wounds with unexplored and/or nonenteric fistulas, and poorly debrided wounds.

Although NPWT may not be the definitive wound management in the other scenarios listed, there is no contraindication for its use.

How well did you know this?
1
Not at all
2
3
4
5
Perfectly
35
Q

A 40-year-old man with a history of seizure disorder presents with jaw pain and malocclusion 12 hours after he was punched in the face during a brawl. X-ray studies show a displaced left angle fracture and right parasymphyseal fracture. He smokes one pack of cigarettes daily. Open reduction and internal fixation of the fractures is planned. Which of the following factors in this patient’s clinical presentation places him at greatest risk for postoperative wound infection?

A) History of seizures
B) History of tobacco use
C) Patient age
D) Patient gender
E) Surgery delayed until 5 days after injury

A

The correct response is Option B.

Although early treatment of mandible fractures would help relieve pain, delay of treatment in multiple series has not been directly correlated with an increased wound infection rate. In a recent review, tobacco use has been associated with a sixfold increase in wound infection compared with nonsmokers. Although the incidence of complications after mandible fractures is lower in children, in the adult population, patient age and gender do correlate with an increased complication rate. Factors that have correlated with a higher complication rate include the number of mandible fractures present and incision location, intraoral and combined intraoral, and combined intraoral and extraoral incisions were associated with higher infection rates than only extraoral incisions.

How well did you know this?
1
Not at all
2
3
4
5
Perfectly
36
Q

Which of the following technical factors has the greatest favorable impact on the final appearance of a surgical scar?

A) Closing the wound in a single layer
B) Use of an absorbable suture
C) Use of topical cyanoacrylate
D) Retention suture
E) Wound-edge eversion

A

The correct response is Option E.

The two technical factors that increase the likelihood of a “good” scar are placement of sutures that will not leave permanent suture marks and wound-edge eversion. In wounds where the skin is brought precisely together, there is a tendency for the scar to widen. In wounds where the edges are everted or hypereverted in an exaggerated fashion, this tendency is minimized possibly by reducing the tension on the closure. While the most common method of closing a wound is with sutures, there is nothing necessarily superior about sutures or a specific type of suture. Staples, skin tapes, or wound adhesives are also useful in certain situations. Regardless of the method of closure or type of suture used, precise approximation of skin edges without tension is essential to ensure healing with minimal scarring. Simple interrupted suture is the gold standard for suturing wounds closed and everting the skin edges. Retention sutures tend to leave the most obvious and unsightly cross-hatching if they are not removed early. Wounds deeper than the skin are closed in layers. The key is to eliminate the dead space and provide a strong closure to prevent dehiscence and reduce tension. However, not all layers necessarily require separate closure.

How well did you know this?
1
Not at all
2
3
4
5
Perfectly
37
Q

The use of routine systemic antibiotic prophylaxis is indicated in which of the following procedures?

A) Abdominoplasty
B) Carpal tunnel release
C) Excision of squamous cell carcinoma of the skin
D) Reduction mammaplasty
E) Rhytidectomy

A

The correct response is Option D.

Systemic antibiotic prophylaxis is recommended in clean breast surgery. Studies have shown that the use of antibiotic prophylaxis in patients undergoing breast surgery (with or without implant) significantly reduces the risk of surgical site infections. The benefit from routine antibiotic prophylaxis is greater in individuals receiving tissue expanders or breast implants for reconstruction, but patients undergoing breast augmentation or reduction mammaplasty also benefit from antibiotic prophylaxis. With the exception of cosmetic breast surgery, clean operations have not been shown to benefit from routine antibiotic prophylaxis. Therefore, the use of routine antibiotic prophylaxis is not indicated in clean surgical cases of the hand (carpal tunnel release), skin (squamous cell carcinoma of the skin), head and neck, or abdominoplasty. It is indicated in contaminated surgery of the hand or face.

How well did you know this?
1
Not at all
2
3
4
5
Perfectly
38
Q

A 45-year-old woman undergoes breast reconstruction with a transverse rectus abdominis musculocutaneous (TRAM) flap. During donor site closure, a size-0 absorbable suture that maintains the greatest strength over time is desired for closure of Scarpa fascia. Which of the following sutures is most appropriate?

A) Chromic gut
B) Poliglecaprone (Monocryl)
C) Polydioxanone (PDS)
D) Polyglactin (Vicryl)
E) Polypropylene (Prolene)

A

The correct response is Option C.

Chromic gut is an absorbable natural monofilament suture whose strength decreases to 50% in approximately 14 days, with near complete loss of strength at approximately 3 weeks.

Polyglactin (Vicryl) is an absorbable synthetic polyfilament suture whose strength decreases to 50% in approximately 2 to 3 weeks, with near complete loss of strength at approximately 1 month.

Poliglecaprone (Monocryl) is an absorbable synthetic monofilament suture whose strength decreases to 50% in approximately 7 to 10 days, with near complete loss of strength at approximately 3 weeks.

Polydioxanone (PDS) is an absorbable synthetic monofilament suture whose strength decreases to 50% in approximately 4 weeks, with near complete loss of strength at approximately 6 weeks.

Polypropylene is a a nonabsorbable synthetic monofilament suture.

How well did you know this?
1
Not at all
2
3
4
5
Perfectly
39
Q

A patient presents with an 8-cm linear laceration to the forearm from a bicycle accident. Compared with typical suturing techniques, which of the following outcomes is more likely with a cyanoacrylate glue–only closure?

A) Dehiscence
B) Hyperpigmentation
C) Infection
D) Keloid scarring
E) Pain

A

The correct response is Option A.

While cyanoacrylate glue closures such as Dermabond and Indermil offer the advantages of speed, ease-of-use, and comfort in the closure process, some studies show that the outcomes are unpredictable, especially for longer lacerations. One pediatric groin hernia incision closure showed a 24% dehiscence rate, while a porcine study of 10-cm lacerations showed a 15% dehiscence rate. Glue closures do have a role in smaller, tension-free lacerations, particularly in children or others who may not easily tolerate traditional closure. This simplicity of closure does come with the cost of a higher dehiscence rate, so glue closures may be inappropriate for longer, more complex wounds. When used as an adjunct to a comprehensive subdermal interrupted closure, it appears that the dehiscence rate normalizes. Data are less convincing on long-term scar results, but it does not appear likely that glue closures improve or worsen scarring to any appreciable extent for wounds that achieve primary healing without disruption.

How well did you know this?
1
Not at all
2
3
4
5
Perfectly
40
Q

A 24-year-old man comes to the office because of a 1-year history of a lesion of the left earlobe following an attempt to pierce his ear at home. A photograph is shown. The patient reports that the lesion is enlarging gradually in size and mildly itchy. Which of the following features most likely characterize this lesion?

A) It is a hypertrophic scar because it extends beyond the margin of the initial scar
B) It is a hypertrophic scar with distinct bundles of collagen oriented parallel to the skin surface
C) It is a keloid and has an increased ratio of type I to type III collagen compared with normal scars
D) It is a keloid and has increased levels of transforming growth factor beta (TGF-B) and type III collagen
E) It is a keloid with distinct bundles of collagen oriented parallel to the skin surface

A

The correct response is Option D.

All wounds leave scars. Wound healing is mediated by fibroblasts and is divided into three phases: the inflammatory phase (3 to 10 days), fibroblastic (10 to 14 days), and maturation (14 days to one year). During the proliferative phase, immature collagen fibrils are cross-linked and form mature collagen. The amount of collagen in a healing wound reaches a peak in three weeks, but remodeling continues months to years after the initial injury.

The difference between a keloid and a hypertrophic scar is often made clinically. A hypertrophic scar does not transgress the margins of the original wound; a keloid invades the adjacent normal skin. From the history of an ear piercing and the appearance of the lesion, the lesion is clearly a keloid.

Normal skin contains distinct collagen bundles that run parallel to the epithelial surface. The collagen structure in disordered scars such as hypertrophic scars and keloids is present in randomly oriented dense sheets.

Keloid scars contain excess amounts of type III collagen. In normal skin and scars the ratio of type I collagen to type III collagen is 4:1. In keloids the ratio is decreased due to excess type III collagen deposited by fibroblasts.

How well did you know this?
1
Not at all
2
3
4
5
Perfectly
41
Q

Negative pressure wound therapy with a sponge dressing is CONTRAINDICATED in which of the following clinical scenarios?

A) An abdominal wound with an enteric fistula
B) A dorsal hand wound with an exposed tendon
C) A lower extremity wound with acute osteomyelitis
D) Over a closed surgical incision
E) A radiated scalp wound with exposed bone

A

The correct response is Option C.

The use of negative pressure therapy (NPT) is contraindicated in wounds with active infection including osteomyelitis. Negative pressure dressings in these wounds convert an open, draining wound into a closed wound, which could potentially lead to abscess formation and/or sepsis. NPT has become an integral part of wound management over the past decade and a half. It is a commonly used wound dressing and/or chronic wound management tool. It is instrumental in acute wounds as well (e.g., lower extremity trauma, abdominal wall trauma), and as a skin graft bolster dressing. The major contraindications for its use include wounds with active infection such as untreated osteomyelitis, malignant wounds, wounds with exposed major vessels and/or organs, and wounds with unexplored and/or nonenteric fistulas.

Apart from the infected wound, all the wounds mentioned in the option set may benefit from the use of NPT. It may not be the definitive management for those wounds, but it could be used as a temporary measure prior to the definitive treatment. Recently, studies have shown that the use of NPT over closed incisions may reduce the risk for dehiscence and infections.

How well did you know this?
1
Not at all
2
3
4
5
Perfectly
42
Q

A 63-year-old man underwent resection of a chest wall sarcoma that was covered with an anterolateral thigh flap. He now undergoes external beam radiation therapy, and there is erythema, edema, and dry desquamation of the surgical sites. Which of the following is the most appropriate treatment?

A) Diphenhydramine
B) Hyaluronic acid
C) Hydrocortisone
D) Salicylate
E) Vitamin E

A

The correct response is Option C.

Radiation dermatitis is one of the most common side effects of radiotherapy for cancer and can occur any time, from hours to weeks after radiation exposure.

Acute radiation-induced skin changes depend on the radiation dose and include erythema, edema, pigment changes, epilation, and dry or moist desquamation. They can also be accompanied by pain and pruritis.

For the lower grade changes described for this patient, topical corticosteroids with low to medium potency, such as hydrocortisone 1% cream, are recommended. This is in addition to a skin-washing protocol to keep skin clean and dry before treatments.

All of the other topical agents listed – antihistamines, salicylate analgesics, vitamin E, and hyaluronic acid – have all been previously studied and shown to have no added benefit.

More severe skin changes such as moist desquamation, skin necrosis, or intractable pain may require radiation to be stopped, resulting in inadequate disease treatment.

How well did you know this?
1
Not at all
2
3
4
5
Perfectly
43
Q

A 38-year-old unconscious and intoxicated woman is brought to the emergency department after being struck by a motor vehicle. She sustained multiple injuries, including a wound on the right thigh, which measures 12 × 18 cm with areas of exposed fat and muscle. There is dirt and gravel in the wound. Which of the following is the most appropriate next step in management?

A) Broad-spectrum antibiotic therapy
B) Injection of tetanus toxoid
C) Negative pressure wound therapy
D) Split-thickness skin grafting
E) Wound irrigation and debridement

A

The correct response is Option B.

In a patient with a grossly dirty wound, it is appropriate to administer a tetanus shot. Tetanus (also known as lockjaw) is characterized by a prolonged contraction of skeletal muscle fibers. The primary symptoms are caused by tetanospasmin, a neurotoxin produced by Clostridium tetani, a gram-positive, rod-shaped, obligate anaerobe. Infection generally occurs through wound contamination and often involves a cut or deep puncture wound.

In the acute period before definitive wound closure can be achieved, it is critical to debride all devitalized tissue such that there is a healthy, viable wound bed. The administration of broad-spectrum intravenous antibiotics has not been shown to decrease the risk of wound infection, and may, in fact, lead to the development of selecting out for resistant pathogens. The mainstay of treatment is performing repeat surgical debridement as often as necessary until the wound is clean. Debridement and cleansing of the wound are ideally performed in the operating room under controlled conditions; however, depending upon the condition of the patient, concomitant injuries, and the ability of the patient to tolerate the procedure, it may be necessary to perform a limited, conservative wound washout at the bedside or in the emergency department. Over the course of the initial hospital stay, debridement should continue until healthy tissue is encountered, which can be identified by visual inspection and the presence of punctate bleeding.

The surgeon must consider several things when deciding between closure with a flap or a graft. The defect in this patient is too large to achieve primary closure. The use of negative pressure wound therapy for such a large wound may be helpful as a temporary measure, but, as a method of definitive wound closure, would result in healing by secondary intention, scarring, and prolonged wound care. If there were exposed bone, tendon, nerves, blood vessels, or significant dead space, this would make a stronger argument for a flap-over-skin graft. Although not provided as an option in this question, the use of biosynthetic materials or dermal matrix tissues has been reported in the literature as an intermediate step to skin grafting, but it is important to consider the necessity of these materials in effecting outcomes in light of the significant cost of using them.

Split-thickness skin grafts can provide wound coverage over a large area. A mechanical dermatome is often used. Typical thicknesses may range from 8/1000th of an inch to 14/1000th of an inch. The graft can be meshed in various ratios such as 1:1.5, 1:2, and 1:3 to allow for a larger area of coverage per unit of harvested skin. It is important that the underlying wound bed be viable and free of necrotic tissue or infection to allow for healing of the skin graft (“skin graft take”). Adequate immobilization of a skin graft is important for “take” of the graft, and can be achieved with negative pressure wound therapy, or tie-over bolster dressing. The thigh has an abundant amount of soft tissue and muscle, which is why skin grafts are often sufficient for wound coverage rather than flaps.

How well did you know this?
1
Not at all
2
3
4
5
Perfectly
44
Q

An 82-year-old man is referred for reconstruction of the scalp after Mohs micrographic surgery for an aggressive squamous cell carcinoma. He is scheduled to undergo radiation therapy as soon as possible after reconstruction. The patient has pulmonary fibrosis and is receiving oxygen via nasal cannula. Physical examination shows a vertex scalp defect of 4 × 4 cm with calvarium exposed throughout. Which of the following is the most appropriate management?

A) Bilaminate neodermis
B) Delayed reconstruction
C) Local tissue rearrangement
D) Radial forearm free flap
E) Split-thickness skin graft

A

The correct response is Option C.

In any reconstruction, many factors (local, regional, and systemic) have to be considered before deciding on a proper treatment option. Indeed, there may very well be multiple options. The patient described is an elderly man who is an extremely poor candidate for anesthesia (example of systemic consideration). Additionally, he will need radiation therapy to the scalp as soon as possible. Therefore, the reconstruction option needs to have excellent blood supply to heal in the first place, heal quickly, and withstand the effects of radiation. Additionally, coverage of the exposed calvarium is necessary, as periosteum has been removed by the Mohs surgeon. Out of the options given, local scalp flap coverage best accomplishes this goal.

Delay of treatment is not recommended because it only creates a greater problem after radiation therapy, because all local options as well as the calvarium will be irradiated, which severely hampers the surgeon’s ability to provide a low-morbidity procedure and avoid a substantial operation.

Dermal matrices (any form) are not appropriate options here for many reasons. They are not the definitive treatment option in a patient who is about to undergo radiation. Once the matrices become incorporated, they will usually need a skin graft to complete reconstruction or they will need a prolonged period of dressing changes, neither of which is optimal in this patient (a second surgery or a prolonged healing phase). Also, they need to be placed on a well-vascularized bed in order for them to “take” and heal more effectively. An exposed calvarium (without additional burring of bone) is not an optimal bed for a dermal matrix.

Skin grafts (of any variety) lack blood supply after harvest. They also need to be placed on a well-vascularized bed in order for them to “take” and heal more effectively. An exposed calvarium (without additional burring of bone) is not an optimal bed for a skin graft. Also, a thin skin graft may not be the best form of reconstruction in a patient who is about to undergo radiation therapy, if other options exist.

Radial forearm free flap is too complex an operation for this patient with many comorbidities and a relatively small defect.

How well did you know this?
1
Not at all
2
3
4
5
Perfectly
45
Q

An otherwise healthy 25-year-old woman is scheduled to undergo resection of a 3 × 5-cm atypical nevus of the right thigh. Medical history includes systemic lupus erythematosus. She has been receiving oral corticosteroid therapy for more than 5 years. She is well nourished. Perioperatively, administration of which of the following vitamins is most likely to improve this patient’s wound healing?

A) A
B) B complex
C) C
D) D
E) E

A

The correct response is Option A.

Perioperative administration of vitamin A is most likely to improve wound healing in this well-nourished patient receiving chronic corticosteroid therapy.

Corticosteroids have been shown to negatively affect all major steps of the wound healing process. Several mechanisms have been proposed, including a stabilizing effect in the lysosomal membrane of cells. Vitamin A was known to cause the opposite (destabilizing) effect in lysosomal membranes in vivo, which led to the first studies on the interaction between these two classes of drugs in wound healing in the 1960s. Since then, supplementation of vitamin A in patients receiving corticosteroids has been shown to counteract most of the deleterious effects of corticosteroids in wound healing, with the exception of wound contraction and infection.

There is no consensus in dose and duration of treatment. Most proposed regimens include oral administration of 10,000 to 25,000 international units (IU) per day for 5 to 14 days. Various topical regimens have also been proposed, with doses around 200,000 IU every 8 hours.

Administration of vitamins B, C, D, or E has not been shown to significantly improve wound healing in well-nourished patients. Vitamin C deficiency impairs collagen synthesis, which may lead to poor wound healing and scurvy.

How well did you know this?
1
Not at all
2
3
4
5
Perfectly
46
Q

A 30-year-old man is evaluated one year after undergoing vascularized allograft transplantation at the midforearm level. Surveillance angiogram and duplex ultrasound show 60% closure of the ulnar artery and complete occlusion of the radial artery. The patient is adherent to the prescribed immunosuppressive therapy. Clinical evidence of chronic ischemia is suspected. Which of the following underlying processes is most likely in this patient?

A) Deposition of preformed immune complex and complement activation
B) IgA-mediated response
C) IgM and IgG antigen–mediated response
D) T-cell–modulated immune response

A

The correct response is Option D.

Hypersensitivity reactions are divided into four different responses.

Type 1 (allergy) refers to immediate release of IgE, mediated release of histamine, and other vasoactive mediators resulting in manifestation within minutes. Examples include asthma or anaphylaxis.

Type 2 (cytotoxic-antibody dependent) refers to binding of IgM or IgG to the target cell, which in this case is a host cell. This results in the membrane attack complex (MAC) destruction of the targeted cell. Examples include thrombocytopenia, Goodpasture, and membranous nephropathy.

Type 3 (immune complex–mediated reaction) refers to IgG binding to circulating antigen resulting in formation of an immune complex. These complexes can end up collecting in the vasculature, joints, and kidneys resulting in local destruction. Examples include rheumatoid arthritis, systemic lupus erythematosus, and serum sickness.

Type 4 (delayed type hypersensitivity) refers to the activation of TH1 helper T cells by an antigen-presenting cell. This establishes an immune response memory and when activated again, the TH1 cells activate a macrophage-mediated response resulting in cellular damage. Examples include chronic transplant rejection, contact dermatitis, and multiple sclerosis.

Transplant patients require immunosuppression to avoid a type 4 hypersensitivity. Although an overwhelming majority of the recent upper extremity transplants have done well, there have been several cases of vascular compromise attributed to chronic rejection. Close monitoring of vascular status is part of the vascularized composite transplant protocol.

How well did you know this?
1
Not at all
2
3
4
5
Perfectly
47
Q

A 32-year-old man with a history of self-inflicted gunshot wound is evaluated because of significant facial deformity despite multiple complex reconstructive procedures. Composite tissue allotransplantation is performed. One episode of rejection is successfully treated 4 weeks postoperatively. Three months postoperatively, the patient develops recurrent swelling and hyperemia of the facial skin. Which of the following is the most likely cause of this condition?

A) ABO incompatibility
B) Acute rejection
C) Antibody incompatibility
D) Chronic rejection
E) Hyperacute rejection

A

The correct response is Option B.

The most likely diagnosis is acute rejection, because this patient is still in the early postoperative period when acute rejection is most likely to occur (0 to 3 months). ABO incompatibility and antibody incompatibility would result in hyperacute rejection, which is mediated by the humoral immune system and occurs within minutes of transplantation. Chronic rejection occurs after years and is characterized by vasculopathy and fibrosis.

How well did you know this?
1
Not at all
2
3
4
5
Perfectly
48
Q

A 36-year-old man with traumatic injuries, who is intubated and sedated in the intensive care unit, is noted to have extravasation of concentrated calcium solution from a peripheral access intravenous line. The consult is made immediately after extravasation. Which of the following is the most appropriate management of this injury?

A) Intravenous administration of dexrazoxane
B) Local injection of hyaluronidase
C) Phentolamine infiltration
D) Topical application of dimethyl sulfoxide
E) Topical application of heat

A

The correct response is Option B.

Hyaluronidase is an enzyme that breaks down hyaluronic acid, a mucopolysaccharide that is a normal component of the interstitial fluid barrier. It has been shown to increase the rate of absorption of an injected substance by facilitating diffusion of the substance over a large area. When injected locally within 1 hour of extravasation, it breaks down hyaluronic acid and decreases the viscosity of the extracellular matrix, and facilitates absorption and dispersal of the extravasated chemical.

The ischemic effects of extravasated vasoconstrictive agents such as norepinephrine and dopamine may be reversed with local infiltration of phentolamine, which is an alpha-blocking agent. Topical heat application has been recommended in vinca alkaloid extravasation to promote local circulation and speed up clearance of the extravasated agent. Topical cooling in animal models has been demonstrated to increase ulcer formation.

Dexrazoxane has been shown to antagonize the effects of several topoisomerase II poisons such as anthracycline agents, including doxorubicin. Recent clinical trials in Europe have demonstrated its efficacy in minimizing tissue damage from anthracycline extravasation if administered intravenously within 6 hours of extravasation. It is now the recommended initial treatment of anthracycline extravasation, especially in light of its FDA approval in 2007.

Dimethyl sulfoxide (DMSO) is a free radical scavenger and an effective solvent. It may also have antibacterial, anti-inflammatory, and vasodilatory properties. Its topical application is effective in preventing ulcerations caused by doxorubicin extravasation.

How well did you know this?
1
Not at all
2
3
4
5
Perfectly
49
Q

A 30-year-old woman who underwent uneventful abdominoplasty is evaluated 2 weeks postoperatively because of midline wound dehiscence with tissue necrosis. She reports that she did not stop smoking before surgery as instructed. A photograph is shown. Which of the following mechanisms is the most likely cause of the delayed wound healing?

A) Decreased catecholamine production
B) Decreased hemoglobin concentration
C) Decreased leukocyte function
D) Increased fibrinogen production
E) Increased microvascular vasoconstriction

A

The correct response is Option E.

Cigarette smoking is a leading cause of preventable death and disability in the United States. Over the past 20 years, several studies have demonstrated an increased risk of postoperative complications following plastic surgical procedures, including rhytidectomy, breast reconstruction, digital replantation, muscle flaps, and body-contouring procedures. Tobacco smoke is a complex mix of particulate matter, volatile acids, and gases. There are over 4000 different compounds in cigarette smoke, many of which are toxic, mutagenic, and carcinogenic. Tobacco-induced vasoconstriction is mediated directly and indirectly by nicotine, a colorless, odorless, and poisonous alkaloid.

Increased cellular levels of nicotine cause direct microvascular vasoconstriction. Indirect pathways of vasoconstriction include the enhancement of thromboxane A2 and stimulation of catecholamine release. Random skin flaps such as abdominoplasty, rhytidectomy, and mastectomy flaps are predominantly supplied by the subdermal plexus, which is very sensitive to sympathomimetic agonists such as catecholamines.

Smoking also increases carboxyhemoglobin levels, which shifts the oxygen-hemoglobin saturation curve to the left. The net result is decreased oxygen-carrying capacity by direct competitive inhibition from carbon monoxide. Other effects caused by smoking include decreased prostaglandin I2 (prostacyclin) production, increased platelet aggregation and blood viscosity, decreased collagen production, decreased red blood cell deformability, increased fibrinogen production, and decreased leukocyte function (mediated by hydrogen cyanide). The net effect is a prothrombogenic state with impaired inflammation that also contributes to slow wound healing. Although fibrinogen production is increased and leukocyte function is decreased, the primary mechanism by which wound healing is impaired is related to the nicotine-induced vasoconstriction of the subdermal plexus.

Rhytidectomy patients who smoke are 12.5 times more likely to develop skin necrosis compared with patients who do not smoke. One study showed a 47.9% rate of wound-healing problems in abdominoplasty patients who smoked compared with 14.8% in those who did not smoke. Another large study of patients undergoing breast reconstruction using a free transverse rectus abdominis musculocutaneous (TRAM) flap showed no difference in free flap survival in those patients who smoked, but the smoking population had a significantly higher rate of mastectomy skin flap loss, abdominal donor-site complications, and hernias. Current recommendations for smokers who desire elective cosmetic surgery are to avoid smoking and all nicotine products for 4 weeks before and after surgery.

50
Q

A 45-year-old man sustains a facial laceration and develops a keloid scar. Compared with a hypertrophic scar, this patient’s scar is most likely to have which of the following characteristics?

A) Decreased fibroblast density
B) Increased fibroblast proliferation rates
C) Increased ratio of type III to type I collagen
D) Regression of the scar over time
E) Smaller and thinner collagen fibers

A

The correct response is Option B.

Hypertrophic scars generally arise during the first few weeks following the initial scar, grow rapidly, and then regress. On the other hand, keloid scars appear later following the initial scar, and then gradually proliferate, often indefinitely.

Both keloid and hypertrophic scars demonstrate increased fibroblast density.

Keloid scars demonstrate increased fibroblast proliferation rates compared with hypertrophic scars.

Keloid scars demonstrate a decreased ratio of type III to type I collagen. This is not observed in hypertrophic scars.

Keloid scars demonstrate thicker, larger, and more randomly oriented collagen fibers compared with hypertrophic scars.

51
Q

A 10-year-old boy underwent removal of a pigmented nevus from his scalp 2 weeks ago with suture closure. The tensile strength of the incision line today is most likely which of the following percentages of its final strength?

A) 10%
B) 20%
C) 40%
D) 60%
E) 80%

A

The correct response is Option A.

The tensile strength of a skin incision 2 weeks following repair is approximately 10%. Classic studies by Madden and Peacock showed that a cutaneous wound achieves 5% of its ultimate strength after 1 week, 10% after 2 weeks, 20% after 3 weeks, 40% after 4 weeks, and 80% after 6 weeks. The scar has its full strength 12 weeks after repair.

52
Q

An 87-year-old woman with a history of squamous cell carcinoma on the left lower extremity comes for evaluation because of the ulcer shown in the photograph. When the tumor did not resolve 9 months ago, she underwent radiation therapy for 4 weeks followed by excision. All margins were negative. Coverage of the wound with a split-thickness skin graft 6 months ago was not successful. Physical examination shows an ulcerated area over the anterior compartment. There is moderate fibrinous debris within the ulcer. Which of the following is the most likely underlying cause of the impeded wound healing?

A) Decreased vascularity
B) Elevated oxygen tension
C) Enhanced angiogenesis
D) Fibroblast hyperplasia
E) Peripheral margin hypokeratosis

A

The correct response is Option A.

Radiation therapy produces many changes in the skin, whether it is directed at the skin, such as for skin cancer, or directed at deeper structures. Direct damage to blood vessels in the wound bed (obliterative endarteritis) produces decreased oxygen tension. Unlike nonirradiated wounds, radiated wounds do not respond with increased angiogenesis. Decreased breaking strength of radiated wounds is caused by both edema of collagen bundles and direct injury to the fibroblasts that would otherwise repair them. Radiated wounds have hyperkeratotic edges, which impair both contraction and keratinocyte migration.

53
Q

A 60-year-old woman is seen in the hospital for a pressure ulcer in the lumbar region. A photograph is shown. A sponge for negative pressure wound therapy is about to be applied directly to the wound. Which of the following is the most likely complication of this therapy in this patient?

A) Enterocutaneous fistula
B) Excessive bleeding
C) Excessive wound drainage
D) Infection
E) Retained sponge in wound

A

The correct response is Option D.

Infection due to retained necrotic tissue would be the most likely complication in this patient. The vacuum-assisted negative pressure wound closure device should not be used in place of good wound care principles such as debridement.

Use of negative pressure wound therapy has been used for pressure ulcers, open abdomen, traumatic extremity wounds, chest wounds, burns, and skin grafts. Negative pressure wound therapy works through mechanisms that include fluid removal, drawing the wound together, microdeformation, and moist wound healing. Several randomized clinical trials support the use of negative pressure wound therapy in certain wound types. Serious complications include bleeding and infection.

Negative pressure wound therapy devices should be used with caution in infected wounds. They should not be used until the wounds are adequately debrided. This wound has not been adequately debrided and negative pressure wound therapy should not be used until necrotic tissue has been removed.

Bleeding is the next most common complication, but is usually seen in anticoagulated patients and after debridement. Use of a conventional gauze dressing for several hours after a debridement before placing a sponge-based negative-pressure wound therapy device may decrease the risk of excessive bleeding. Most significant bleeding has occurred secondary to disruption of major vessel grafts, cardiac bypass grafts, or the ventricle itself when sponges are placed directly on the structures. This wound is not near any major blood vessels.

Use of a single sponge or a long roll of gauze within any deep wounds is recommended to avoid retained foreign bodies.

Negative pressure wound therapy has been used to control wound drainage. Increased drainage would be caused by the lack of debridement and infection.

Even in clean wounds, a recent report on abdominal wound closure found the most likely complication to be infection rather than recurrent hernia or enterocutaneous fistula. This wound is on the back and would not be likely to have an enterocutaneous fistula. Although initially contraindicated for use with enterocutaneous fistula, recent reports have shown its use to be safe and effective in selected cases.

54
Q

A 56-year-old woman who has been undergoing treatment for breast cancer has pain around the port site 6 hours after the extravasation of paclitaxel from a subcutaneous tunneled subclavian vein catheter. The patient is hemodynamically stable and breathing comfortably. Moderate swelling and tenderness are observed between the port and clavicle. Which of the following is the most effective management?

A) Application of calcium gluconate gel
B) Application of topical collagenase
C) Line change over a wire
D) Line removal and observation
E) Operative debridement

A

The correct response is Option D.

This patient has paclitaxel extravasation due to a malpositioned or leaking catheter with minimal symptoms; therefore, removal of the line and observation is warranted. Calcium gluconate gel is indicated after generously washing areas exposed to hydrofluoric acid as it neutralizes the fluoride ion. Topical collagenase is indicated in wounds with limited tissue necrosis and thus has no role in this patient. Changing this patient’s line over a wire is contraindicated as the catheter is either malpositioned or broken. Although operative debridement is sometimes indicated in extravasation injuries, it is unusual, and expectant management is the norm. As this patient has no acute signs of compartment syndrome or tissue necrosis, line removal and observation are indicated.

The incidence of extravasation is 0.01 to 6%. Chemotherapeutic agents that cause reactions are classified as irritants or vesicants. Irritants cause immediate and typically limited local reactions such as erythema, warmth, and tenderness. Common irritants are: bleomycin, carboplatin, carmustine, cisplatin, dacarbazine, etoposide, ifosfamide, and thiotepa. Vesicants can cause erythema, blistering, and skin necrosis. Itching in the absence of pain is common. In addition, vesicants can cause delayed ulceration that is self-perpetuated when the vesicant is rereleased upon lysis of affected cells. Common vesicants are: dactinomycin, daunorubicin, epirubicin, idarubicin, mechlorethamine, mitomycin, mitoxantrone, paclitaxel, vinblastine, vincristine. Paclitaxel is derived from the bark of the Pacific yew tree and induces microtubular assembly and stabilization, which leads to cell death. It is a vesicant, and if extravasation occurs, symptoms can range from localized pain, swelling, and erythema to severe skin necrosis and ulceration requiring surgical debridement. The vast majority of extravasations are managed non-operatively.

55
Q

A 55-year-old woman who is wheelchair-bound has a stage IV ischial pressure ulcer. She has a history of systemic lupus erythematosus and multiple sclerosis. Medications include prednisone and gabapentin. BMI is 21 kg/m2 and has been stable for the past year. White blood cell count is 10.5 × 109/L, hematocrit is 30%, and serum albumin concentration is 3.6 mg/dL. After debridement of nonviable tissue, wound care is instituted. Supplementation with which of the following is most likely to promote wound healing?

A) Echinacea
B) Ferrous gluconate
C) Glutamine
D) Lipid emulsion
E) Vitamin A

A

The correct response is Option E.

Vitamin A is essential because it promotes epithelialization in collagen synthesis for wound healing, and supplementation is advocated in patients on chronic corticosteroid immunosuppressive medications such as prednisone. A 20,000-IU daily dosage can be useful for wound healing in immunosuppressed or irradiated patients and appears to reverse the wound healing–suppressive effects of the medication.

Patients with chronic wounds frequently have some form of malnutrition that can impede the wound-healing process. In this case, the patient has a serum albumin concentration within the reference ranges, and a stable BMI, signifying adequate protein. In protein-deprived patients, supplementing amino acids that serve as the building blocks of protein synthesis is vital. L-arginine, in particular, has been shown to augment wound healing and collagen production. One study in elderly human subjects found that daily supplementation of 30 g of arginine aspartate for 14 days resulted in markedly enhanced collagen production and total protein.

Ferrous gluconate is a useful supplement in iron deficiency anemia. This patient has borderline anemia, though not of a severity likely to be the central impediment to wound healing. Echinacea is a common herbal supplement used as an immunostimulant but has also been shown to have immunosuppressive effects. Lipid emulsion would be useful in a severely malnourished patient, though in this case, the patient’s BMI is stable in the normal range. Of note, omega-3 fatty acids appear to inhibit the quality of collagen strength, and avoiding this common supplement during healing may be advisable.

56
Q

A 73-year-old man is evaluated for a non-healing wound on the medial aspect of the calf. The wound has been present for 8 months, and he has undergone several months of serial debridements and moist wound care without improvement. A photograph is shown. Ten years ago, he was diagnosed with squamous cell carcinoma of the medial calf skin, and the condition was managed solely with radiation therapy. Which of the following is the most appropriate next step in management?

A) Hyperbaric oxygen therapy
B) Negative pressure wound therapy
C) Wound biopsy and culture
D) Wound debridement and skin graft
E) Continued observation and wound care

A

The correct response is Option C.

Based on the clinical scenario described, wound biopsy and culture is the most appropriate management option. Despite wound debridement and moist wound care, the wound has not improved and is in the region of a previous malignancy. Wound biopsy would allow the diagnosis of recurrent malignancy and aid in the determination of further surgical intervention. Wound culture would allow the diagnosis of soft-tissue infection contributing to the wound’s persistence.

Although wound debridement would be beneficial in this case, application of a skin graft in the face of possible recurrent malignancy and probable marked radiation injury would be associated with increased risk of delayed wound healing and may delay management of recurrent malignancy. If the wound was attributed only to radiation therapy, a better strategy would be to excise the irradiated soft tissues and cover the whole defect with a well-vascularized flap.

Hyperbaric oxygen therapy has been shown to be beneficial for the management of radiation soft-tissue injury. This therapeutic modality should only be instituted after a complete evaluation of the patient’s wound, which would include soft-tissue biopsy because the patient previously had a malignancy in the region.

Complete evaluation of the wound would include pertinent history and physical examination, evaluation of the patient’s nutritional status, examination of extremity vascular inflow and outflow, diagnosis and treatment of wound infection, and optimization of wound characteristics.

The patient has already undergone debridement and wound care for several months; therefore, continued observation and wound care would be an inadequate management option.

It is inappropriate to perform negative pressure wound therapy in an irradiated wound without diagnosis by tissue biopsy.

57
Q

A 29-year-old man comes to the office because of scarring 12 weeks after he sustained extensive chemical burns to 30% of the total body surface area. Examination shows thick hypertrophic scarring of the upper extremities and anterior torso. Which of the following is the most appropriate management?

A) Injection of a corticosteroid
B) Scar band revision
C) Serial casting
D) Topical application of vitamin E
E) Use of pressure garments

A

The correct response is Option E.

Compression decreases blood flow to active scars, leading to decreased production of collagen fibers. This results in a balance of collagen synthesis and lysis that produces a flatter, softer, less vascularized scar. Clinically, burn scar hypertrophy is managed by use of pressure garments and inserts that must be worn almost 24 hours per day. They should be initiated as soon as all burn wounds have closed enough to tolerate wear and continued until the burn scar has matured. Initially, the pressure applied is low (15 to 17 mmHg). Then, as the scar progresses in maturation, custom-made pressure garments that provide 24 to 28 mmHg of pressure may be fabricated for the patient.

The prompt institution of splinting techniques after the acute phase of burn injury can limit the development of long-term deformities. Splinting can combat edema, protect exposed structures and balance soft-tissue lengths to prevent contracture formation and compensate for functional deficits. Later, during the remodeling phase, serial casting can be a great adjunct to a therapeutic exercise program to restore normal range of motion. Surgical lengthening and scar band revision are options that are evaluated if hypertrophic scarring and contractures still develop after appropriate rehabilitation and management.

Although the depth and distribution of the injury factor into the development of scars, the patient’s own genetic predisposition also plays a role in scar formation and maturation.

Injection of a corticosteroid can improve hypertrophic scars, but its use is limited to small, focused areas. Metabolic effects can be considerable. Due to the extent of scarring in this patient, corticosteroids are not an appropriate option.

Although other topically applied therapies, such as creams containing vitamin E, have been widely used with the intent to improve wound healing, there is not substantial evidence to support regular use. Thirty-three percent delayed hypersensitivity reaction can be seen with topical vitamin E.

58
Q

A 33-year-old African American woman has a large recurrent keloid of the left earlobe. Reexcision with postoperative radiation therapy is planned. Which of the following is the most likely long-term complication of this therapeutic plan?

A) Altered pigmentation
B) Desquamation
C) Itching
D) Skin cancer
E) Telangiectasia

A

The correct response is Option A.

The patient described has a recurrent keloid after previous excision. Surgery alone has recurrence rates of over 50%, and combination therapies including injection of a corticosteroid, pressure earrings, and surgery can have marked recurrence rates.

For recurrent keloids, post-excision radiation therapy, usually given in one to three fractions, has efficacy rates between 6 and 98%. The most common long-term complications of radiation therapy include hypo- or hyperpigmentation (62%) and telangiectasias (27%). Skin desquamation is an acute reaction to radiation therapy and occurs in 24% of patients. Secondary malignancies after radiation therapy for keloids are very rare. Itching from keloids is usually improved with treatment.

59
Q

A male newborn is evaluated because of the scalp anomaly shown in the photograph. Which of the following is the most appropriate initial management of the affected area?

A) Application of a skin substitute
B) Local wound care with antibiotic ointment
C) Primary closure
D) Skin grafting
E) Tissue expansion

A

The correct response is Option B.

This child has aplasia cutis congenita, or cutis aplasia, of the scalp. First described in 1767 by Cordon, cutis aplasia is the congenital absence of all skin layers including the epidermis, dermis, and subcutaneous fat. This process most commonly affects only focal areas of tissue but involvement can be extensive. The majority of cases involve the scalp, but this process can occur in any cutaneous area of the body. Cutis aplasia can occur in isolation or as part of a syndrome, the most common being Adams-Oliver syndrome. Cutis aplasia of the scalp can range from small areas of involvement that often heal in utero and appear at birth as a “congenital scar” to massive defects that are devoid of scalp and cranium. Most small- or intermediate-sized full-thickness defects heal quickly (as in the patient described) if kept moist and the resultant scar can be excised secondarily. Bone healing is often complete in small lesions, and residual defects can be reconstructed when the child is older if needed. Large areas are more problematic and extensive scalp defects that threaten dural integrity may require early operative intervention. Cutis aplasia involving large areas of the scalp has a reported mortality ranging from 20 to 55%, typically as a result of sagittal sinus hemorrhage or associated congenital defects. In such cases, coverage of the dura can be life-saving. Described methods of soft-tissue coverage include skin graft, cultured allograft, acellular dermis, and immediate or delayed reconstruction with a flap. Tissue expansion of the scalp in a newborn presents many challenges and is not recommended.

60
Q

A 12-year-old boy is brought to the emergency department with a soft-tissue injury to the left knee after falling while playing football. Which of the following types of cells is most likely to appear first at the wound site?

A ) Fibroblast
B ) Lymphocyte
C ) Macrophage
D ) Neutrophil
E ) Platelet

A

The correct response is Option E.

The process of wound healing occurs as a sequence of overlapping processes. The appearance of cell types in an acute wound occurs in the following order: platelets, neutrophils, macrophages, lymphocytes, and fibroblasts, during the inflammatory phase.

Tissue injury causes injured vessels to constrict rapidly, with primary hemostasis being a platelet-mediated process. Platelets trapped in the clot contain growth factors that initiate the coagulation and wound-healing cascade.

The ensuing phases of wound healing consist of inflammation, collagen synthesis, angiogenesis, epithelialization, and remodeling.

During the inflammatory phase, after platelet aggregation and degranulation, chemoattractants, activation factors, and vasoconstrictors are released. An efflux of neutrophils occurs at the wound site to primarily sterilize the wound. Within 2 to 3 days, the inflammatory cell population shifts to monocytes that differentiate into macrophages, which orchestrate the repair process. Collagen synthesis occurs as circulating bone marrow-derived cells migrate into the wound and develop a fibroblastic cell function. These cells and local, activated fibroblasts synthesize and secrete the replacement collagen scar. Fibroblasts become the predominant cell type by 3 to 5 days in clean, noninfected wounds. As fibroplasia progresses, granulation tissue forms as a consequence of neoangiogenesis and the directed growth of vascular endothelial cells stimulated by platelet and activated macrophage and fibroblast products. Wound reepithelialization occurs as keratinocytes at the wound margins migrate and proliferate once epidermal continuity is reestablished. Remodeling of the resultant scar is a dynamic process that occurs slowly over months to years. Collagen deposition and degradation occur to yield a mature scar; however, maximum tensile strength of a wound reaches only approximately 80% of noninjured skin.

61
Q

A 50-year-old man with a history of organ transplantation is scheduled to undergo resection of a squamous cell carcinoma of the scalp followed by reconstruction with a flap. This patient is most likely to avoid postoperative wound-healing complications if he is currently undergoing which of the following immunosuppressive therapies?

A ) Antilymphocyte antibody (basiliximab)
B ) Antimetabolite (azathioprine)
C ) Calcineurin inhibitor (cyclosporine)
D ) Glucocorticosteroid (prednisone)

A

The correct response is Option A.

Many immunosuppressive agents used in organ transplantation have been shown to impair wound healing. Thus, free tissue transfer or major reconstructive surgery has been associated with higher complication rates. Immunosuppressive agents can be categorized as antilymphocytes (lymphocyte immune globulin [Atgam], thymoglobulin, basiliximab), antimetabolites (azathioprine, mycophenolate mofetil), calcineurin inhibitors (cyclosporine, FK-506), and glucocorticosteroids. Only antilymphocyte therapy has been shown not to impair wound repair.

62
Q

Which of the following characteristics best distinguishes keloid scar tissue from hypertrophic scar tissue?

A ) Collagen fibers parallel to the direction of wounding
B ) Extension beyond original scar
C ) Improved by surgical excision alone
D ) Increased fibroblast density
E ) Location on flexor surfaces and areas of motion

A

The correct response is Option B.

Keloid scars differ from hypertrophic scars in that they can extend beyond the original scar, whereas hypertrophic scars are confined to the original boundary.

Collagen fibers are wavier in keloids and more parallel in hypertrophic scars. Light and electron microscopic studies demonstrate that collagen in keloids is disorganized compared with normal skin. The collagen bundles are thicker and wavier, and the keloids contain hallmark ?collagen nodules? at the microstructural level.

Surgical excision alone has a high rate of recurrence for keloids.

Increased fibroblast density occurs in both hypertrophic scars and keloid scars and cannot be used to differentiate between the two. Keloids have increased fibroblast proliferation rates.

Hypertrophic scars commonly occur on flexor surfaces and joints. Keloids have a high predilection for the sternum and earlobe.

63
Q

A 41-year-old man undergoes an elective transplantation of the right hand 2 years after traumatic amputation in a machine accident. Postoperatively, the patient takes immunosuppressive medications to minimize the chance of rejection. To monitor for cellular rejection, observation and biopsy of which of the following tissue types in the postoperative period is most appropriate?

A ) Blood vessel
B ) Bone
C ) Muscle
D ) Skin
E ) Tendon

A

The correct response is Option D.

Composite tissue allotransplantation (CTA) has been performed on a host of tissues, though more recently in plastic surgery; this has largely been in the field of hand or upper extremity and facial transplantation. This requires immunosuppressive regimens which have had varying degrees of success, as well as issues with patient compliance, especially as these medications are expensive and, at least at this time, necessary for the rest of the patient’s life. Skin is thought to be the most antigenic and immunoreactive tissue in CTA. Experience from China in hand transplantation demonstrated that cellular rejection in these patients was largely limited to the skin, with relative sparing of the underlying blood vessels, bone, muscle, nerve, and tendon. However, as the skin is an easily monitored tissue (versus solid organs), it is the most sensitive indicator of acute rejection in that it is clearly visible and can be easily evaluated by both patient and physician. Therefore, this tissue type is most appropriate to be monitored and biopsied.

64
Q

A 17-year-old boy undergoes excision of a congenital nevus of the scalp. Prior to excision, he underwent placement of a subgaleal tissue expander. Which of the following growth factors is most likely to be upregulated during ischemia in this patient?

A) Epidermal
B) Keratinocyte
C) Platelet-derived
D) Transforming
E) Vascular endothelial

A

The correct response is Option E.

Vascular endothelial growth factor (VEGF) is an important mediator of wound healing and is necessary for angiogenesis. It was originally discovered as a protein secreted by tumor cells to increase the permeability of local blood vessels to circulating macromolecules. It has been shown to increase endothelial growth and migration and enhance glucose transport in the endothelial cell, which is needed to match the increased energy required during angiogenesis. Hypoxia has been shown to be a potent stimulus for the expression of VEGF, and current research has been directed at utilizing VEGF to augment healing and viability in situations of tissue ischemia.

The remaining growth factors are all important in the wound-healing process, relating primarily to reepithelialization and wound contraction, but are not directly involved in angiogenesis.

65
Q

A 10-year-old girl is referred to the office because of a large, full-thickness cranial defect after sustaining a traumatic injury. Reconstruction with a split cranial bone graft is performed. Which of the following is the most likely mechanism by which the bone graft heals?

A) Dural ossification
B) Osteoconduction
C) Osteogenesis
D) Osteoinduction
E) Vasculogenesis

A

The correct response is Option B.

The most likely mechanism of split cranial bone graft healing is osteoconduction. The split cranial bone graft is primarily cortical. After it is separated from its blood supply, it serves as a nonviable scaffold for the ingrowth of blood vessels and osteoprogenitor cells from the recipient site. This process of osteoconduction, or ?creeping substitution,? eventually leads to resorption and replacement of most of the graft with new bone. The graft becomes fully osseointegrated with the recipient site.

Spontaneous dural ossification can heal full-thickness cranial defects in infancy. After 12 to 18 months of age, the dura will not spontaneously ossify.

Osteogenesis is the primary mechanism of bone graft healing for cancellous or vascularized bone grafts. Because these grafts are revascularized rapidly, osteoblasts survive the transplantation and produce new bone at the recipient site.

Osteoinduction involves the stimulation of mesenchymal cells at the recipient site to differentiate into bone-producing cells. Demineralized bone and bone morphogenetic protein produce new bone primarily by osteoinduction.

Vasculogenesis, the de novo formation of blood vessels from precursor cells, occurs during embryogenesis. Revascularization of split cranial bone graft occurs by angiogenesis, the production of new vessels from preexisting vasculature.

66
Q

A 24-year-old woman comes to the office because of painful nodules in both buttocks. She underwent buttock augmentation with injections of liquid silicone by an unlicensed practitioner 4 years ago. Excision of the affected area is performed. Histology of a specimen obtained from the excised tissue is most likely to show which of the following?

A) Acellularity
B) Calcification
C) Granuloma
D) Necrosis
E) Thrombosis

A

The correct response is Option C.

Free silicone liquid has a long history of use for soft-tissue augmentation. Little regulation of the practice and variable degrees of purity of the silicone have resulted in many disastrous complications, often occurring years after the initial injections. Potential adverse sequelae following silicone injection include migration, chronic induration and pigmentary changes, painful subcutaneous nodules, chronic infection, and ulceration. Many of the treated areas require radical resection and reconstruction.

Histologic study of postsilicone injection nodules typically shows granulomas which develop after initial inflammation and fibrosis. Histologic evaluation of typical capsule formation around solid alloplastic prostheses, including breast prostheses, shows acellularity and organized layers of collagen. In the breast, free silicone injection may result in ductal obstruction, which may appear as calcification on mammography. Necrosis may be noted in ulcerative-type complications seen in intradermal injection. Intravascular injection can result in thrombosis and, rarely, embolism, resulting in death.

67
Q

A 63-year-old man comes for evaluation of a dehisced surgical incision 3 weeks after undergoing open reduction of the right ankle. He has a history of coronary artery disease, hypertension, hypercholesterolemia, and poorly controlled type 2 diabetes mellitus. He had a myocardial infarction 2 years ago. Physical examination shows a dehisced surgical incision with exposed tibialis anterior tendon without paratenon. A photograph is shown. The patient refuses free tissue transfer. Which of the following is the most appropriate skin substitute for the wound?

A) Biodegradable bilaminate neodermal matrix (Integra)
B) Biosynthetic wound dressing (Biobrane)
C) Cryopreserved neonatal fibroblast-derived dermal substitute (Dermagraft)
D) Human fibroblast-derived composite skin substitute (TransCyte)
E) Living bilayered skin substitute (Apligraf)

A

The correct response is Option A.

Integra is a bilaminate neodermal replacement product that is composed of a biodegradable bovine collagen-glycosaminoglycan (collagen-GAG) matrix underlayer with a silicone outer layer. Although its ?on-label? indication is for burn reconstruction, it also has utility in reconstruction of wounds of exposed bone without periosteum, exposed cartilage without perichondrium, and exposed tendon without paratenon, such as in the scenario described. The collagen-GAG matrix serves as scaffolding for the ingrowth of cells and neovascularization. After regeneration, which takes between 2 to 4 weeks, the silicone outer later is removed and a thin split-thickness skin graft completes the reconstruction by providing epithelial cells over the neovascularized dermal replacement.

Biobrane is a temporary, rather than permanent, bilaminar skin substitute that is constructed of an inner layer, composed of nylon and collagen, which is covered by an outer silicone film. Biobrane serves as a temporary wound dressing, usually in burn patients, where it helps prevent evaporative loss (due to the silicone outer layer) and subsequent wound desiccation. It decreases wound pain and provides a barrier to bacterial infection. Biobrane is removed either before permanent grafting or after epithelialization of the wound has occurred. Dermagraft is a dermal substitute composed of neonatal foreskin fibroblasts cultured on a polyglactin mesh, and it is generally used in the treatment of diabetic foot ulcers, where it often is combined with meshed skin grafts.

TransCyte is also a temporary wound dressing. It is similar to Biobrane but has an added biologic layer derived from neonatal fibroblasts that are seeded onto the nylon matrix to produce type I collagen, fibronectin, and glycosaminoglycans. TransCyte is removed either before skin grafting or after epithelialization of the wound. It has been shown to significantly decrease pain and time to epithelialization.

Apligraf, another permanent replacement product, is constructed of type I bovine collagen and cultured neonatal human fibroblasts and keratinocytes. After construction of the dermal matrix equivalent, cultured keratinocytes are applied. It is generally used in the treatment of venous ulcers and diabetic foot ulcers (may take more than one application). Its long-term durability, however, makes it an inappropriate choice in situations with a full-thickness defect with exposed vital structures.

A follow-up photograph of the ankle is shown.

68
Q

A 67-year-old woman comes to the office for follow-up examination 6 months after debridement of a chronic nondiabetic wound to the lower leg. Following the procedure, the patient was treated with moist dressings. Physical examination shows that the wound is healing less than 15% weekly. Persistent bacteria are suspected. Application of which of the following is the most appropriate nonsurgical management at this time?

A) Alginate dressings
B) Collagenase
C) Film or transparent dressings
D) Hydrogel dressings
E) Silver ion-impregnated dressings

A

The correct response is Option E.

Silver ions kill a broad spectrum of bacteria. No resistant organisms have been identified, and it is nontoxic to human cells. Alginates absorb up to 20 times their weight and are used to exudate wounds. Films and transparent dressings are waterproof and would be impermeable to bacterial contamination. Hydrogels are generally waterproof and would prevent bacterial contamination.

69
Q

Epithelial cell migration across an acute skin laceration is initiated by which of the following mechanisms?

A ) Contraction of myofibroblasts

B ) Deposition of collagen into the wound

C ) Formation of a fibrin-fibronectin plug

D ) Loss of contact inhibition

E ) Secretion of anti-inflammatory products

A

The correct response is Option D.

Epithelial cell migration is initiated by loss of contact inhibition and occurs from the periphery of the wound and adnexal structures. Cell division occurs in 48 to 72 hours, resulting in a thin epithelial cell bridge across the wound. A key role is played by epidermal growth factors.

Myofibroblasts are involved in wound contraction and play no role in epithelialization. Collagen deposition is seen in the remodeling phase of wound healing. Fibronectin produced by fibroblasts serves as an adhesion molecule anchoring cells to collagen or proteoglycan substrates. Release of cytokines from platelets plays an important role in the initiation of the hemostatic initial phase.

70
Q

A 65-year-old woman is evaluated because of nonhealing sores on her lower extremities. She has a history of alcoholism and is homeless. She appears cachectic, pale, and severely malnourished. She has lost most of her teeth; the gums are purplish and spongy in appearance. Skin examination shows numerous petechiae. Large, superficial, nongranular sores are noted on the legs. Scurvy is suspected. Which of the following processes is most likely to be adversely affected by this patient’s nutritional deficiency?

A ) Collagen cross-linking

B ) DNA synthesis

C ) Epithelialization

D ) Fibroblast proliferation

E ) Immune modulation

A

The correct response is Option A.

The patient described most likely has a vitamin C deficiency. Vitamin C is an essential nutrient for collagen cross-linking via the hydroxylation of proline and lysine to hydroxyproline and hydroxylysine, respectively. The lack of cross-linking results in impaired collagen synthesis and a decrease in collagen tensile strength. Collagen-containing tissues, such as skin, dentition, bone, and blood vessels, are therefore affected, leading to the development of scurvy. The hallmark signs of scurvy are hemorrhaging in any organ (ie, petechiae, swollen gums), loss of dentition, and a lack of osteoid formation. Deficiency of vitamin C is rare in the United States; however, it can be seen in patients who are severely malnourished; have a history of alcoholism; or have restrictive diets for medical, social, or economic reasons.

Other nutrients also play a major role in healing. Folate and vitamin B6 (pyridoxine) are integral in DNA synthesis and cellular proliferation. Vitamin A is an essential factor in epithelialization and fibroblast proliferation. Vitamin E is a strong antioxidant and immune modulator. Zinc is one of the most important micronutrients, as it acts as a cofactor for numerous metalloenzymes and proteins. It is essential for proper protein (like collagen) and nucleic acid synthesis.

71
Q

During which of the following phases of wound healing is the net rate of collagen synthesis greatest?

A ) Contraction

B ) Fibroblastic

C ) Hemostasis

D ) Inflammatory

E ) Remodeling

A

The correct response is Option B.

During the fibroblastic phase, three to five days after injury, fibroblasts migrate into the wound and lay new collagen. Type 3 collagen predominates early in normal wound healing but is later replaced by Type 1 collagen. It is during this time that the greatest rate of collagen synthesis occurs in the wound.

Contraction begins during the fibroblastic stage (proliferative) and continues well into the remodeling phase.

Hemostasis is the brief period before healing starts. This allows for vasoconstriction and clotting to be activated.

Collagen synthesis does not occur during the inflammatory phase; instead, there is activation of the inflammatory system to allow €œcleansing € of the wound in preparation for healing.

During the remodeling phase (maturation), the net amount of collagen is constant as there is an equal amount of degradation and synthesis. However, the wound becomes stronger because there is cross-linking of the collagen fibers. Maximum tensile strength is achieved after approximately 12 weeks.

72
Q

A 62-year-old man is scheduled to undergo repair of an abdominal wall hernia. A preoperative photograph is shown. He has a 15-year history of chronic lung disease and receives daily corticosteroids to control his symptoms. Supplementation with which of the following is most likely to decrease impairment of wound healing in this patient?

A ) Vitamin A

B ) Vitamin B1 (thiamine)

C ) Vitamin B2 (riboflavin)

D ) Vitamin B6 (pyridoxine)

E ) Vitamin C

A

The correct response is Option A.

Animal studies have shown that impairment of wound healing caused by use of corticosteroids can be reversed by the oral administration of vitamin A (retinoic acid), 15,000 IU daily for seven days. Patients with true deficiencies of vitamin B6 (pyridoxine), vitamin B1 (thiamine), or vitamin B2 (riboflavin) may have wound healing problems and benefit from supplementation. Vitamin C deficiency (scurvy) results in inability to cross-link collagen fibers and decreased wound tensile strength, which may be reversed by vitamin C supplementation.

73
Q

An 11-year-old girl has full-thickness dermal necrosis in the infusion zone of an antebrachial intravenous catheter which was used for treatment of a metastatic lower extremity sarcoma. Localized swelling was noted five days earlier after approximately 100 mL of the medication extravasated into the subcutaneous tissues. Total parenteral nutrition and intravenous doxorubicin had been administered as well as intravenous cefazolin and vancomycin. CT of the leg with intravenous contrast medium had also been performed in the past week. Which of the following is the most likely causative agent of the dermal necrosis from extravasation injury?

A ) Cefazolin

B ) Doxorubicin hydrochloride

C ) Radiographic contrast medium

D ) Total parenteral nutrition

E ) Vancomycin

A

The correct response is Option B.

Extravasation injuries are potentially dangerous occurrences that necessitate careful clinical follow-up and early treatment to avoid late catastrophic sequelae. Cytotoxic and hyperosmolar agents may result in local tissue necrosis, and high-volume injuries may cause compartment syndrome and limb ischemia. Though such injuries can occur in any patient, higher risk groups include children and the elderly, intensive care and chronically ill patients.

While any of the agents listed could be harmful in sufficient volume, doxorubicin hydrochloride (Adriamycin) is the one agent that should raise particular alarm. Adriamycin is associated with severe soft-tissue necrosis and warrants close follow-up for early surgical debridement, if needed. Dilution of the agent with saline or hyaluronidase may be helpful. Other early interventions, which are standard to all extravasation injuries, include splinting, elevation, local dressings, and close serial examination.

74
Q

An 80-year-old woman has been receiving papain-urea ointment for management of a pressure necrosis wound over the lateral aspect of the right calf. A 4 * 10-cmdiameter zone of black, dry, insensate eschar is noted on the right calf. A 1-cm zone of mild erythema and slight liquefaction of the eschar edges are noted. No tenderness to palpation is noted. Pedal pulses are present. Which of the following is the main disadvantage of using a papain-urea ointment in this patient?

A ) Elevation of compartment pressures

B ) Inadequate debridement

C ) Painful dressing changes

D ) Renal toxicity

E ) Resistant organism infection

A

The correct response is Option B.

The patient described has a full-thickness dermal injury with ensuing liquefactive necrosis that requires sharp debridement to healthy tissue. Enzymatic ointments are not sufficient for this level of necrotic tissue burden. Papain is a potent digestant of nonviable protein material but does not affect healthy tissues. Urea increases the digestive potency of papain. Collagenases are another class of commonly used enzymatic debridement agents.

The patient described has no clinical evidence of compartment syndrome, and enzymatic debridement would not elevate compartment pressures. Presence of a life- or limb-threatening condition, including compartment syndrome, fasciitis, deep space infection, or ischemia, would necessitate surgical intervention rather than continued care with local dressings.

In some patients, these ointments can cause burning and they may be diluted with a hydrogel to reduce pain. Dressings containing silver ions inactivate papain and they should not be used together. Although burning is a drawback, the main disadvantage in the scenario described is the inability of these agents to treat large areas with a significant amount of necrosis. These are slow-acting agents that allow separation of superficial eschar over days to weeks. They are useful in smaller wounds in patients who may not easily tolerate or who are logistically unable to undergo surgical debridement.

Renal toxicity has not been identified with papain-urea.

Although the patient described is at risk of colonization by resistant organisms, infection is not a contraindication of enzymatic debridement.

75
Q

A 52-year-old woman (shown) is evaluated six years after bilateral mastectomies and radiation therapy for cancer of the left breast because of a new, small open area near the left axillary fold which she first noted three weeks ago. She has been compliant with postoperative oncologic surveillance. Temperature is 98.9 °F (37.2 °C), pulse is 80 bpm, respirations are 16/min, and blood pressure is 140/75 mmHg. Physical examination shows a 2 * 2-cm open ulcer on the left chest wall with exposed rib. Which of the following is the most likely cause of the ulcer?

A ) Abscess

B ) Loss of skin integrity from intertriginous shearing forces

C ) Lymphedema drainage tract

D ) Osteoradionecrosis of the underlying ribs

E ) Recurrent breast cancer

A

The correct response is Option D.

The effects of ionizing radiation are permanent and may present either acutely or in delayed fashion, even years after the original radiation insult. The mechanism of injury from this radiation is through free radical production which, in turn, directly damages the DNA. In the acute period, the effects of radiation may manifest themselves as erythema and edema of the skin, vasodilation with endothelial edema, and lymphatic obliteration. This eventually leads to capillary thrombosis and subsequent inadequate tissue oxygenation. Over time, nonhealing ulcers can spontaneously develop, sometimes years later.

Abscesses usually would present initially with pain, erythema, and localized fluctuance and are often with associated fever and/or malaise.

Intertriginous shearing would most often present as superficial epidermal loss with possible superinfection with yeast due to moisture.

Lymphedema can be a chronic condition after mastectomy and axillary node dissection and is usually manifested as generalized edema of the ipsilateral upper extremity. Sinus tract formation is rare.

Although recurrent cancer is always a concern in patients with a personal history of cancer, proper, regular, and thorough surveillance can often detect recurrences early, especially in compliant patients. Most recurrences occur within the first five years.

76
Q

A 42-year-old man develops a dehiscence of the abdominal incision six weeks after undergoing a lower body lift. Medical history includes a 100-lb (45-kg) weight loss during the past three years. Which of the following is the most likely cause of the wound-healing problem?

A ) Hematoma

B ) Patient movement

C ) Seroma

D ) Skin necrosis

E ) Wound infection

A

The correct response is Option C.

Body lift procedures after massive weight loss have a complication rate of approximately 50%. The most common complication is wound dehiscence, which occurs in greater than 30% of patients. Wound dehiscence can be characterized as either early (in the immediate postoperative period) or late. Early wound dehiscence may be caused by patient movement, while late wound dehiscence is often due to underlying seroma. Although infection and skin necrosis can occur in the postoperative period and result in wound dehiscence, seroma is much more common.

77
Q

A 76-year-old woman with type 1 diabetes mellitus is scheduled to undergo surgical intervention for chronic ulceration of the lower extremities. Which of the following factors is NOT likely to impair wound healing in this patient?

A ) Advanced age

B ) Chronic anemia

C ) Chronic use of corticosteroids

D ) Malnutrition

E ) Poor control of diabetes

A

The correct response is Option B.

A number of local and systemic factors have been shown to impair wound healing. Diabetes mellitus adversely affects healing by altering circulation, attenuating inflammation, reducing tissue oxygenation, and adversely affecting glucose metabolism resulting in stress hyperglycemia. Malnutrition, including caloric, protein, vitamin, and mineral insufficiency, impairs the immune system, prevents tissue repair, and may lead to progression or recurrence of a wound. Aging is associated with reduced production of collagen and angiogenesis and a diminished response to environmental stresses. By reducing inflammation, steroids impair angiogenesis, fibrogenesis, wound contraction, reduced wound strength, and delay healing. Other factors such as infection, smoking, poor tissue oxygenation, radiation, chemotherapy, and the presence of foreign bodies or cancer within a wound are also associated with poor healing. Anemia, even to severe levels, when circulation is maintained has not been found to impair wound healing.

78
Q

A 26-year-old man comes to the emergency department because he has a laceration of the anterior aspect of the right lower leg. Physical examination shows a superficial 2-cm full-thickness skin laceration. Sutures are placed. If the wound heals normally, which of the following is the earliest time that the epidermis is likely to be restored?

(A) 12 Hours

(B) 24 Hours

(C) 2 to 3 Days

(D) 4 to 5 Days

(E) 6 to 7 Days

A

The correct response is Option B.

If the basement membrane has been destroyed, epithelial cells and keratinocytes located on wound edges proliferate and send out projections to reestablish a protective barrier against fluid loss and bacterial invasion. The stimuli for epithelial proliferation and chemotaxis are epidermal growth factor and transforming growth factor (TGF) €‘α produced by activated platelets and macrophages. Fibroblasts do not synthesize TGF €‘α.

After closing a surgical incision, epithelialization usually occurs within 24 hours, at which point it is no longer necessary to keep the wound dry. Washing to remove dried blood can reduce bacterial proliferation and improve wound healing. This process may take longer in patients in whom wound healing may be compromised, such as elderly patients or patients with diabetes.

Epithelialization occurs early in wound healing. If the basement membrane remains intact, epithelial cells migrate upward in the normal manner. The epithelial progenitor cells remain intact below the wound in skin appendages, and the normal layers of the epidermis are restored in two to three days.

79
Q

In an acute wound, which of the following structures initiate coagulation, hemostasis, and the inflammatory cascade?

(A) Endothelial cells

(B) Eosinophils

(C) Macrophages

(D) Neutrophils

(E) Platelets

A

The correct response is Option E.

Initial changes in a wound after injury are vascular in nature. Blood vessels are disrupted and hemorrhage ensues, damaging the epidermal barrier. After vasoconstriction occurs, the coagulation cascade is activated to reduce blood loss. Platelets in the clot are essential for hemostasis and normal inflammatory response. Platelets release adenosine diphosphate (ADP), which, in the presence of calcium, stimulates further platelet aggregation. Alpha granules in the platelets release cytokines such as platelet €‘derived growth factor (PDGF), transforming growth factor (TGF)-β, TGF-α, basic fibroblast growth factor (bFGF), platelet factor IV, and β €‘thromboglobulin. These proteins initiate the wound healing cascade by attracting and activating fibroblasts, endothelial cells, and macrophages. Platelets also contain lysosomes and dense bodies in their cytoplasm. Lysosomes include proteases. Dense bodies store vasoactive amines, such as serotonin, which increase microvascular permeability. The extrinsic and intrinsic coagulation pathways are activated, resulting in a fibrin mesh with aggregated platelets embedded in it.

The early inflammatory phase following coagulation activates complement and initiates the classic molecular cascade, which leads to infiltration of the wound with neutrophils within 24 to 48 hours of injury. Neutrophils are attracted to the site of injury via chemical messengers released by damaged tissue, platelets, bacteria, and inflammatory mediators. Neutrophils act as defensive units, phagocytosing bacteria and foreign debris from the wound to prevent infection. Neutrophils are phagocytosed by macrophages and destroyed.

Macrophages are key regulatory cells for repair. When circulating monocytes migrate through the blood vessel wall and into the wound, they transform into macrophages. Between 48 and 72 hours after injury, macrophages are the dominant cells in the wound. They function in phagocytosis of bacteria and dead tissue. Macrophages also secrete collagenases and cytokines responsible for proliferation of fibroblasts, resulting in collagen production, and for proliferation of endothelial cells, resulting in angiogenesis.

Eosinophils do not play a role in acute wound healing.

80
Q

A 55-year-old woman is admitted to the hospital for treatment of chronic pancreatitis. She has a 10-year history of severe rheumatoid arthritis managed with corticosteroids. Physical examination performed on admission shows an ulcer of the right ischium with purulent drainage. Results of culture show a polymicrobial infection. Serum albumin level is 1.8 g/dl. Necrotic soft tissue is debrided, resulting in a 6 × 4-cm defect and exposure of the underlying ischium. Which of the following is the most appropriate next step in management?

(A) Enzymatic debridement

(B) Vacuum €‘assisted closure (VAC) therapy

(C) Skin graft

(D) Gluteus fasciocutaneous flap

(E) V €‘Y hamstring advancement flap

A

The correct response is Option B.

The patient described has a full-thickness wound with exposed bone and will be a good candidate for flap closure once her infection is resolved and her nutrition optimized. Immediate reconstruction in a malnourished patient increases the risk of wound dehiscence and infection. During the interim, a vacuum €‘assisted closure (VAC) device is the most appropriate coverage for the wound. The VAC device promotes wound healing by facilitating the removal of excess interstitial fluid due to an increased pressure gradient and causes mechanical deformation of the wound resulting in enhanced granulation tissue formation, even over bone.

Enzymatic debridement may be appropriate in some patients with pressure sores but is not required in this patient because she has already undergone surgical debridement. A skin graft will not take to bone and provides insufficient soft-tissue coverage.

81
Q

Under optimal conditions, the peak tensile strength of a skin incision is achieved at approximately how many days after injury and reaches what percentage of the tensile strength of unwounded skin?

Days Percentage

(A) 30 75

(B) 60 80

(C) 90 85

(D) 120 90

(E) 150 95

A

The correct response is Option B.

The peak tensile strength of skin is achieved at approximately 60 days after injury and reaches approximately 80% of the original unwounded tensile strength.

Several factors can influence wound healing, including ischemia, anemia, steroids, malnutrition, smoking, age, denervation, foreign bodies, infection, radiation, and other systemic conditions (eg, diabetes, cancer, renal failure).

82
Q

Hyperbaric oxygen therapy has the greatest utility in the treatment of which of the following wounds?

(A) Diabetic foot ulcer with osteomyelitis

(B) Extravasation injuries

(C) Grade 4 pressure sore of the ischium

(D) Pyoderma gangrenosum

(E) Superficial partial-thickness burn

A

The correct response is Option A.

The effect of hyperbaric oxygen (HBO) therapy on wound healing has been shown in several clinical trials. There is proven utility in conditions such as osteomyelitis, necrotizing infections, and ischemia reperfusion injury. The use of HBO in the treatment of diabetic lower extremity wounds has shown improved healing rates and decreased amputations. This would be particularly valuable when conventional therapy has failed and underlying bone and tendons are exposed. HBO has no proven utility in the treatment of extravasation injury, pressure sores, or pyoderma gangrenosum. There is insufficient evidence to recommend HBO in the treatment of burn wounds, although there may be numerous theoretical advantages.

83
Q

A poorly nourished 70-year-old woman is brought to the emergency department after sustaining burns in a house fire. Examination shows partial-thickness burns on 10% of the total body surface area. Nutritional supplementation is planned. Which of the following best describes the role of vitamin C in wound healing?

(A) Acts as a cofactor in the hydroxylation of proline and lysine for procollagen formation

(B) Alters prostaglandin production by inhibiting phospholipase A2 activity

(C) Inhibits leukocyte migration into the wound

(D) Promotes epithelialization and fibroblast proliferation through its effect on metalloenzymes

(E) Promotes formation of oxygen free radicals

A

The correct response is Option A.

Vitamin C plays a pivotal role in collagen synthesis, being an essential cofactor in the hydroxylation of proline and lysine for procollagen formation. Procollagen residues are then altered intracellularly to form collagen. Vitamin C deficiency therefore results in scars of poorer tensile strength and abnormal capillary formation.

In addition, vitamin C has an antioxidant function, neutralizing the effects of oxygen free radicals and can increase resistance to infection by facilitating leukocyte migration into the wound. Alteration of prostaglandin production, by inhibition of phospholipase A2 activity, is a function of vitamin A. Zinc promotes epithelialization and fibroblast proliferation through its effect on metalloenzymes.

84
Q

A 72-year-old man presents with a 24-hour history of a diabetic foot infection. Medical history includes type 1 diabetes. Physical examination shows palpable dorsalis pedis and posterior tibial pulses. Sensation is intact. There is skin necrosis and erythema on the dorsum of the foot between the first and second metatarsals. In addition to early surgical debridement and antibiotic therapy, which of the following interventions decreases the likelihood of below-the-knee amputation?

A) Application of granulocyte colony-stimulating factor
B) Hyperbaric oxygen therapy
C) Topical antibiotic therapy
D) Topical treatment with iodophor gauze
E) Topical treatment with Wonder of the World plant (Kalanchoe pinnata)

A

The correct response is Option A.

There have been numerous studies on diabetic foot infection. With the mainstay of treatment being surgical debridement and systemic antibiotic therapy, multiple adjuncts to treatment have been used and studied. In a systematic analysis of the literature, papers studying multiple modalities were evaluated, including topical antibiotic therapy, hyperbaric oxygen therapy, application of granulocyte colony-stimulating factor, topical treatment with Wonder of the World plant, and using iodophor gauze. Of these treatments, the application of granulocyte colony-stimulating factor was found to reduce the likelihood of below-the-knee amputation.

85
Q

A 21-year-old man sustains a flame burn to the distal aspect of the left forearm, resulting in a hypertrophic scar. Silicone gel sheeting is applied to the scar. Which of the following is the most likely mechanism of action that the silicone gel sheeting will have on the scar?

(A) Alteration of cytokine levels

(B) Direct chemical effect

(C) Hydration

(D) Increased oxygen tension

(E) Pressure

A

The correct response is Option C.

Although the exact mechanism of action of silicone gel sheeting is unknown, the most widely accepted hypothesis is that there is an increase in hydration resulting from occlusion, which is supported by in vitro data. Other studies have either ruled out, or not supported, alteration of cytokine levels, direct chemical effects, increased oxygen tension, or pressure.

It is generally thought that for silicone gel sheeting to be effective it must be worn for at least 12 hours a day for three months or longer.

86
Q

A 3-year-old girl is brought to the office by her parents two months after sustaining an injury to the right ankle for evaluation of the scar shown. Which of the following is the most appropriate management?

(A) Excision

(B) Oral administration of a corticosteroid

(C) Radiation therapy

(D) Topical administration of vitamin E

(E) Observation

A

The correct response is Option E.

This patient has a hypertrophic scar. Correct diagnosis of the abnormal scar will directly influence treatment options for this patient.

Hypertrophic scars and keloid scars are clinically distinct entities with different treatment approaches.

Hypertrophic scars develop soon after the injury (within six to eight weeks). They can worsen up to six months but subside with time. The extent of scarring relates to the initial depth of injury. Hypertrophic scars can produce contractures, especially over joints. The boundaries of the original scar are maintained. Hypertrophic scars have a predilection to occur over the flexor surface of joints.

Keloid scars may develop months after the injury. They seldom regress and are not associated with contractures. The boundaries of the original wound are overgrown, and the extent of the scar can far exceed the original tissue injury. Keloid scars are commonly found on the deltoid, upper back, chest, and earlobes.

Both hypertrophic and keloid scars are raised, erythematous, and often pruritic.

87
Q

A 37-year-old African American woman comes to the office for consultation regarding a 5-cm nodule on the posterior aspect of the left earlobe that has been enlarging over the past three months. There is no history of trauma. Which of the following are the most likely biologic and morphologic characteristics of this patient €™s lesion?

Fibroblast Myofibroblast Blood Vessel
Proliferation Status Density

(A) Increased Absent Decreased

(B) Increased Present Increased

(C) Normal Absent Increased

(D) Normal Present Decreased

(E) Normal Present Increased

A

The correct response is Option A.

The patient described has an earlobe keloid. The distinction between hypertrophic and keloid scars is often difficult to make based on clinical features. Keloids are more frequently associated with more darkly pigmented skin and are commonly seen on earlobes or in the deltoid or presternal region.

Keloids and hypertrophic scars can also be difficult to differentiate histopathologically and biologically. Recent advances in cellular biology have identified several key differences between the two.

The predominant cell of scar tissue is the fibroblast, which is responsible for producing collagens. Various in vitro studies have shown that fibroblasts cultured from keloids have increased proliferation rates and decreased apoptosis when compared with fibroblasts cultured from both hypertrophic scars as well as from normal skin.

While morphologic and immunohistochemical changes occur within a scar as it matures, immature hypertrophic scars express an increased density of blood vessels, whereas keloid scars show a decreased density of blood vessels. Myofibroblasts are present in hypertrophic scars but absent in keloids, which in general, are far less cellular.

Both transforming growth factor (TGF) €‘β1 and TGF €‘β2 have been shown to be expressed in greater levels in fibroblasts from both keloids and hypertrophic scars when compared with those from normal skin.

88
Q

Which of the following laboratory results of fluid analysis is increased in chronic wounds relative to acute wounds?

(A) Growth factor level

(B) Matrix deposition

(C) Metalloproteinase level

(D) Protease inhibitor level

(E) Tissue oxygen tension

A

The correct response is Option C.

Chronic wounds have an interruption in the natural sequence of wound healing involving a highly regulated cascade of events among many cell types, soluble factors, and matrix components. The chronic wound microenvironment is characterized by an imbalance between matrix-degrading enzymes and their inhibitors. Metalloproteinase levels are elevated relative to acute wounds, resulting in extracellular matrix degradation.

In chronic wounds, the healing process is disrupted by a prolonged inflammatory phase. Proinflammatory cytokines are elevated, which leads to an increase in protease activity and a decrease in protease inhibitor and growth factor levels. This results in decreased matrix deposition, which prevents epithelization and healing.

Tissue oxygen tension is abnormally low in the central aspect of chronic wounds.

89
Q

Which of the following is the mechanism of action of pressure garments in management of fibroproliferative scars?

(A) Alteration in cell shape

(B) Hypoxia of local tissue

(C) Increase in synthesis of tissue proteinases

(D) Increase in temperature of the scar

(E) Induction of matrix-specific autoantibodies

A

The correct response is Option B.

The mechanisms of pressure garments in management of fibroproliferative scars include local tissue hypoxia, reduced fibroblast proliferation, and reduced collagen synthesis.

Pressure therapy is a conservative treatment modality that has been used for many years, particularly in the treatment of hypertrophic scars after burn injury. Numerous studies have documented that pressure therapy reduces the size as well as softens hypertrophic scars. The mechanism of action behind pressure garments is believed to be secondary to tissue ischemia. The pressure leads to local hypoxia, which, in turn, decreases tissue metabolism and increases collagenase activity. It also reduces fibroblast proliferation and collagen synthesis. The exerted pressure is effective between 24 and 30 mmHg. Studies have shown a response to pressures as low as 5 to 15 mmHg. At these pressures, the inherent capillary flow, but not the peripheral circulation, is overcome, and there is occlusion of the small vessels within the scar. Pressure therapy should begin as soon as re-epithelialization occurs.

Alteration in cell shape has not been shown to occur with use of pressure garments in management of fibroproliferative scars.

Increased synthesis of tissue proteinases is a mechanism of corticosteroids. When used as a single therapy in fibroproliferative scars, corticosteroid injections have a variable response rate that ranges from 50% to 100%. In addition to softening the scars, corticosteroid injections often provide symptomatic relief of itching and pain.

Increased temperature of the scar has not been proven as a possible effect of topical silicone gel sheeting. Other possible mechanisms of action of silicone are changes in skin hydration and downregulation of wound healing by the negative charge of the silicone.

Induction of matrix-specific autoantibodies is a molecular mechanism that blocks the effect of transforming growth factor (TGF)-β. The inhibition of TGF €‘β may serve as a new approach to scar therapy. Autoantibodies and binding proteins that function against TGF €‘β are still experimental and are not available for routine clinical use.

90
Q

Two months after undergoing reduction mammaplasty, a 28-year-old woman has scars that are softening but maintaining strength. The mechanism by which this process occurs is an increase in which of the following?
(A) Chondroitin-4 sulfate
(B) Hyaluronic acid
(C) Integrin
(D) Type I collagen
(E) Water content

A

The correct response is Option D.

During the maturation phase of wound healing, the formerly indurated, raised, and pruritic scar becomes a mature scar while the wound continues to gain tensile strength. Tensile strength is measured as the maximum tension a material can withstand without tearing. Experimental evidence suggests that collagen fibers are largely responsible for the tensile strength of wounds. Most of the embryonic Type III collagen laid down in early wound healing gets replaced by mature Type I collagen until the normal skin ratio of 4:1 Type I to Type III is reestablished. Hyaluronic acid and chondroitin-4 sulphate levels decrease to resemble those of normal dermis, and the water content of the tissues gradually returns to normal.

91
Q

During the inflammatory phase of wound healing, which of the following cellular components is most likely to appear first?
(A) Fibroblasts
(B) Lymphocytes
(C) Macrophages
(D) Myofibroblasts
(E) Neutrophils

A

The correct response is Option E.

Wound healing begins at the moment that tissue integrity is traumatically disrupted. Platelets are the first cells to enter the wound and provide the first burst of soluble molecules that modulate and mediate an initial hemostatic phase of wound healing. As hemostasis ensues secondary to vasoconstriction, platelet activation, and activation of the clotting cascade, various substances are present in the wound site that subsequently result in secondary vasodilation, increased capillary permeability, and chemoattraction and activation of leukocytes.

Neutrophils are the first leukocytes to enter the wound and thereby establish acute inflammation, peaking at approximately 24 hours post-wounding, followed shortly thereafter by the appearance of macrophages and lymphocytes.

The appearance of fibroblasts, epithelial cells, and endothelial cells characterize the subsequent proliferative phase of wound healing.

92
Q

In creation of a normal collagen molecule, the amino and carboxy terminal peptides must be removed from which of the following molecules?
(A) Collagen fiber
(B) Collagen fibril
(C) Hydroxylated lysine
(D) Procollagen
(E) Proline

A

The correct response is Option D.

Procollagen is the molecule that is secreted from the cells and has its amino and carboxy terminal ends cleaved off to form a collagen molecule. The collagen molecule then can crosslink with other collagen molecules to form a collagen fibril and those fibrils crosslink and weave with other fibrils to become a collagen fiber. Proline and lysine undergo intracellular hydroxylation early in the formation of the procollagen molecule. These processes require many cofactors and are altered in disease processes like Ehlers-Danlos syndrome.

93
Q

One year after ear piercing, a 21-year-old woman has the slow-growing posterior auricular lesion shown. Pathologic examination of this lesion is most likely to show excess of which of the following?
(A) Basal cells
(B) Collagen
(C) Fat
(D) Melanin
(E) Myofibroblasts

A

The correct response is Option B.

The lesion pictured is an earlobe keloid, which is clinically characterized by exuberant growth of proliferative scar outside the boundaries of the initial scar bed. Lesions may develop many months after the initial healing period and rarely regress. The lesion has been associated in certain populations with autosomal dominant inheritance pattern and is much more likely in dark-pigmented individuals (4% to 16%). No gender predominance is known, and the lesions are highly recurrent — in some studies 50% or greater. On histology, no myofibroblasts are noted, unlike hypertrophic scars. The characteristic histologic finding reveals extensive random collagen fibrils in densely packed bundles. The clinical history and lesion as pictured do not represent another skin tumor (basal/squamous cell carcinoma or melanoma) nor do they represent a deeper subcutaneous fatty growth (lipoma).

94
Q

A 62-year-old man with type 2 diabetes mellitus has a nonhealing wound on the right foot six months after he sustained a degloving injury of the dorsal surface of the right foot. Radiographs obtained at the time of injury showed no abnormalities. Physical examination shows a 6 _ 8-cm wound on the dorsal aspect of the foot with minimal granulation tissue, exposed tendons, and intact sensation to the sole. Which of the following is the most appropriate diagnostic study?
(A) Semmes-Weinstein monofilament test
(B) MRI of the foot
(C) Bone scan
(D) Determination of ankle-brachial index
(E) Measurement of transcutaneous oxygen

A

The correct response is Option E.

Not all wounds are capable of spontaneous healing. Patients with arterial insufficiency can experience delayed wound healing. The possibility of spontaneous healing of a wound can be assessed objectively. For a wound to heal spontaneously, the ankle pressure should exceed 40 mmHg, the great toe pressure should be greater than 40 mmHg, the great toe pressure should be greater than 40 mmHg, and the patient should have pulsatile plethysmography and a transcutaneous oxygen higher than 30 torr. If one of these criteria is not met, the patient can be expected to have difficulty healing and further vascular assessment with angiography. Sometimes, these patients require revascularization to heal a wound. A flap reconstruction should not be performed on a patient with arterial insufficiency because this could result in an ischemic flap and nonhealing donor site or recipient vessel site wounds.

Semmes-Weinstein monofilament test will document qualitative sensory function. MRI will demonstrate the volume of soft tissue and bone involvement; a bone scan would indicate possible involvement of osteitis. None of these are determinants of healing. In a diabetic patient with noncompliant arteries, ankle-brachial index probably would be falsely normal.

95
Q

A 32-year-old man undergoes surgical repair of a deep, contaminated laceration of the dorsum of the left hand with general anesthesia and an upper arm tourniquet. Exploration shows laceration of the extensor tendons. Thirty minutes into the procedure, the anesthesiologist reports that prophylactic antibiotics have not been administered. To decrease risk of infection without adversely affecting the healing of the tendons, the most appropriate intervention is initiation of intravenous administration of antibiotics, debridement, and cleansing/irrigation of the wound with a 1:1000 dilution of which of the following solutions?
(A) 0.25% Acetic acid
(B) Normal saline
(C) 3% Hydrogen peroxide
(D) 1% Povidone-iodine
(E) 0.05% Sodium hypochlorite

A

The correct response is Option D.

The mainstay of treatment for contaminated wounds includes the use of prophylactic intravenous antibiotics and mechanical debridement. Prophylactic antibiotics should be administered before inflation of a tourniquet for obvious reasons. In this case, which is an unfortunately common scenario, tissue levels of antibiotics in the hand will be nonexistent during the surgery with the tourniquet inflated. Although antibiotic levels in the wound will increase upon release of the tourniquet, merely ignoring the need for antibiotics in a contaminated wound is not appropriate and administering only antibiotics with the tourniquet inflated is not adequate. Irrigation of wounds with topical antimicrobials has shown efficacy over irrigation with saline alone. However, the concentration of the topical antimicrobial impacts fibroblast toxicity (undesired) and bactericidal activity (desired).

In the choices listed, only irrigation with a 1:1000 solution of 1% povidone-iodine (0.001% povidone-iodine) will provide full bactericidal activity without significant fibroblast toxicity. A 1:1000 solution of 3% hydrogen peroxide or 0.25% acetic acid would be more damaging to fibroblasts than to bacteria. 0.05% Sodium hypochlorite at a 1:1000 dilution is too dilute (1:1000 dilution of 0.05% is 0.00005%). While nontoxic to fibroblasts, 0.00005% sodium hypochlorite is no longer effectively bactericidal. Therefore, in this scenario, the most appropriate management would be to initiate intravenous administration of antibiotics, debride the wound, and irrigate the wound with 0.001% povidone-iodine. Many wound solutions are used; only 0.001% povidone-iodine and 0.005% sodium hypochlorite are toxic to common gram-negative and -positive bacteria and nontoxic to fibroblasts.

96
Q

Which of the following processes of healing provides maximal tensile strength of a wound?

(A) Accumulation of collagen
(B) Addition of sugar moieties
(C) Hydroxylation of lysine
(D) Hydroxylation of proline
(E) Molecular cross-linking

A

The correct response is Option E.

Intramolecular and intermolecular cross-linking between collagen fibers accounts for the maximal tensile strength of a wound. Maximal strength occurs during the remodeling phase of wound healing. Peak increase in tensile strength occurs three to six weeks after injury but approaches maximal after about three months when it achieves up to 80% of the normal skin strength.

Collagen synthesis peaks at about three weeks, and collagen accumulates to its maximum at six weeks; however, intramolecular and intermolecular cross-linking between collagen fibers provides the tensile strength of the wound.

The addition of sugar moieties occurs just before cleavage of amino and carboxy terminal ends. After this, the molecules are termed collagen, which then develops further intermolecular and intramolecular bonds for strength.

The hydroxylation of lysine and proline in the endoplasmic reticulum of the fibroblasts is a crucial step in collagen production and is important in future intermolecular cross-linking. However, this step occurs much earlier in wound healing, primarily during the proliferative phase.

97
Q

Which of the following types of cells has been shown to mediate wound contraction?

(A) Epithelial cells
(B) Lymphocytes
(C) Macrophages
(D) Myofibroblasts
(E) Polymorphonuclear cells

A

The correct response is Option D.

Myofibroblasts, described by Gabbiani in 1971, are thought by most people to mediate wound contraction. They are derived from fibroblasts in the wound, which under conditions of stress elongate and show features of a myocyte. Through interaction with the matrix, they effectively retract collagen fibrils. Various mediators such as transforming growth factor-beta (TGF-_) and platelet-derived growth factor (PDGF) are involved in the process. Myofibroblasts first appear in the wound by the third day after injury and persist for approximately 21 days, after which time they slowly disappear. They persist longer in open contracting wounds.

Epithelial cells are required to cover a wound but play no role in the wound contraction process. Polymorphonuclear cells, lymphocytes, and macrophages are leukocytes involved in the inflammatory response to injury.

98
Q

Which of the following types of collagen is most abundant in a healed scar?

(A) I
(B) II
(C) III
(D) IV
(E) V

A

The correct response is Option A.

The most abundant type of collagen in a healed scar is Type I. This type is the most abundant collagen in the body, including the skin. Type II collagen is found predominantly in cartilage and vitreous. Type III collagen is the second most abundant collagen in a healed scar. It also exists in elastic tissues, such as blood vessels. Type IV collagen is located mainly in the basement membranes. Type V collagen is widespread.

99
Q

Which of the following is the predominant cell responsible for the intermediate phase of wound healing and collagen synthesis (days 3 through 21)?

(A) Erythrocyte
(B) Fibroblast
(C) Myoepithelial cell
(D) Neutrophil
(E) Platelet

A

The correct response is Option B.

The intermediate phase of wound healing begins on the second or third day after injury and continues until approximately 21 days after injury. This phase begins with chemotaxis and proliferation of mesenchymal cells, angiogenesis, and epithelialization. Ultimately, collagen synthesis, wound contraction, and proteoglycan synthesis predominate in this phase; fibroblasts and macrophages are the primary cells involved. Before this phase, the primary effects of wound healing involve hemostasis and inflammation. Initially, the cellular elements involved in this initial phase are erythrocytes and platelets. Neutrophils are the first of the leukocytes found in the area and are mobilized not long after the erythrocytes and platelets. After approximately 21 days, wound remodeling permeates the overall healing environment. This phase is said to end after approximately one year, although wound remodeling is actually a lifelong process.

100
Q

Which of the following interventions is LEAST likely to improve the appearance of a hypertrophic scar?

(A) Application of silicone gel sheeting
(B) Application of vitamin E gel
(C) Intralesional injection of a corticosteroid
(D) Pressure therapy
(E) Prolonged application of paper tape

A

The correct response is Option B.

Application of vitamin E products is popular in the skin-care industry despite the paucity of scientific evidence about its effectiveness. Some animal models have demonstrated improvement in healing of radiation-induced wounds with vitamin E. However, no studies have shown clear-cut improvement in hypertrophic or normal scars. In fact, the only controlled study showed no benefit. Localized dermatitis may occur with application of vitamin E products.

Although various treatments have been used to improve the appearance and texture of hypertrophic scars, no single method has shown uniform success. Response rates greater than 50% are considered successful. Application of silicone gel sheeting has shown significant improvement in fibroproliferative scars in several controlled trials, although the mechanism is unknown.

Intralesional injection of triamcinolone and other corticosteroids typically have a response rate greater than 50% but can cause skin atrophy, depigmentation, telangiectasis, and pain.

Pressure therapy has been used to manage keloids and hypertrophic scars since the early 1970s. The use of pressure garments (specially fitted elastic garments often with silicone inserts) to treat postburn scarring and contractures is a standard of care.

Application of adhesive microporous tape to fresh surgical wounds has been endorsed by an international panel on scar management. Uncontrolled clinical trials have shown its efficacy. The mechanism is unknown but may be similar to the action of silicone gel sheeting.

101
Q

A 16-year-old boy has the scar on the left shoulder shown in the photographs above. What is the minimum recurrence rate of this type of scar following surgical excision only?

(A) 5%
(B) 10%
(C) 25%
(D) 55%

A

The correct response is Option D.

This 16-year-old boy has a keloid on the left shoulder. Unlike hypertrophic scars (which remain within their original boundaries), keloids are abnormal scars that extend beyond the confines of the healing wound and do not regress. Recurrence rates following surgical excision alone have been shown to be at least 55% and as high as 100%, according to the results of some studies. Therefore, excision alone is not recommended; it should instead be combined with postoperative injection of corticosteroids and/or application of gel sheeting or compression garments to minimize recurrence.

In patients with more severe keloids, a short postoperative course of radiation therapy is recommended following excision to decrease recurrence rates to an acceptable level.

102
Q

Which of the following is the predominant cell type involved in wound contracture?

(A) Eosinophil
(B) Erythrocyte
(C) Fibroblast
(D) Monocyte
(E) Neutrophil

A

The correct response is Option C.

Fibroblasts, specifically myofibroblasts, are the predominant cell type involved in wound contracture. These cells first appear approximately three days after injury and are typically located at the periphery of the wound, but contain actin-rich filaments that act throughout the area of injury to initiate contracture and alter the shape of the open wound. Wound contracture is a cell-mediated process that typically begins four to five days after the initial injury and continues until at least 21 days after injury. It can be influenced by many factors, including the degree, area, and shape of the injury and the length of time that the wound remains open. Transforming growth factor-beta and possibly other cytokines may also contribute to wound contracture.

Although erythrocytes, monocytes, and neutrophils are important cell mediators in the wound healing process, they are not primarily involved in wound contracture. Eosinophils are typically involved in hypersensitivity and allergic reactions.

103
Q

Which of the following is the most likely mechanism of action of silicone sheeting/gel pads in enhancing scar maturation?

(A) Decreasing wound tension
(B) Deregulating cellular integrins
(C) Enhancing epidermal contact inhibition
(D) Increasing the static electronegative field
(E) Maintaining regulated wound temperature

A

The correct response is Option D.

Silicone sheeting and silicone gel pads are used to treat hypertrophic or immature scars and keloids. Although their exact mechanism of action is unknown, some surgeons postulate that their positive effect is associated with the generation of an increased static electronegative field by the silicone. This mechanism of action results in favorable wound effects. Other theories propose that the wound-healing mechanism is related to the decreased oxygenation, sustained pressure, or hydrating effects of silicone oil resulting from the use of these products.

Silicone sheeting/gel pads have not been shown to decrease wound tension, affect epidermal contact inhibition, or regulate intracellular integrins or wound temperature.

104
Q

Which of the following is an absolute contraindication to performing vacuum-assisted closure (VAC) therapy for wound management?

(A) Bacterial colonization of the wound
(B) Open fracture of a long bone
(C) Presence of an enteric fistula
(D) Presence of exposed blood vessels
(E) Presence of osteomyelitis

A

The correct response is Option D.

Vacuum-assisted closure (VAC) is an effective technique for management of open wounds. Advantages include promoting the ingrowth of healthy granulation tissue, decreasing the duration of the wound healing process, simplifying dressing changes, and increasing the intervals between dressing changes. However, the presence of exposed arteries or veins is an absolute contraindication to VAC therapy because the vessel may burst and subsequently hemorrhage into the VAC device; this can be potentially fatal.

Although VAC therapy is not contraindicated in open wounds, which by their nature are colonized by bacteria, the presence of gross bacterial infection precludes the use of the VAC device.

VAC therapy is an option for management of open fractures until definitive flap reconstruction can be performed.
The presence of an enteric fistula within the wound is no longer an absolute contraindication to VAC therapy.

The presence of osteomyelitis in the wound bed is not a contraindication to VAC therapy.

105
Q

Which of the following is the ratio of type I collagen to type III collagen in hypertrophic or immature scars?

(A) 1:4
(B) 1:2
(C) 2:1
(D) 4:1

A

The correct response is Option C.

Patients with hypertrophic or immature scars have a type I to type III collagen ratio of approximately 2:1 in the healing wound. In contrast, the type I to type III ratio in normal skin is 4:1.

Type I collagen is present in greater than 90% of the body’s tissues, including bone, tendon, and skin. Type II collagen is predominant in hyaline cartilage and eye tissues. The skin, arteries, uterus, and intestinal wall contain type III collagen, and most fetal wound collagen is type III. Basement membrane is made up predominantly of collagen types IV and V.

106
Q

Deep mechanical massage has been shown to result in which of the following?

(A) Accumulation of collagen bands
(B) Accumulation of mast cell aggregates
(C) Retention of adipocyte cell architecture
(D) Thickening of the epidermis

A

The correct response is Option A.

Deep mechanical massage, using therapeutic massage units (ie, Endermologie), can be performed for reduction or correction of moderate amounts of cellulite and is often used postoperatively in patients who have undergone body contouring procedures. According to the results of an experimental animal study, there is an accumulation of dense longitudinal collagen bands in the middle and deep subcutaneous regions that increases proportionately with the number of treatments administered. Distortion and disruption of adipocytes were also demonstrated in this study.

Deep mechanical massage has not been shown to have any effect on epidermal thickness or accumulation of mast cell aggregates.

107
Q

Which of the following is the primary disadvantage of autologous cartilage grafting?

(A) Immunogenicity
(B) Resorption
(C) Rigidity
(D) Warping

A

The correct response is Option D.

Autologous cartilage grafts are versatile and can be used for joint reconstruction and soft-tissue fill. The grafts can be carved easily; they retain form with minimal resorption. Because autologous cartilage grafts are biocompatible, there is no risk for rejection.

Types of cartilage used for grafting include hyaline, elastic, and fibrocartilage. Hyaline cartilage functions as a covering for the articular surface of bones. The nasal alae and septum, costal cartilage, and trachea are composed of hyaline cartilage. Elastic cartilage is found in the external ear, epiglottis, and portions of the larynx. Fibrocartilage is firm and comprises intervertebral disks, ligaments, and tendons.

The primary disadvantage of autologous cartilage grafting is the potential for warping. There is an inherent tension within the subperichondrial layer that is released when the cartilage is not carved in a balanced cross section.

108
Q

Which of the following factors has been shown to stimulate fibroblasts to produce collagen?

(A) Platelet-derived growth factor (PDGF)
(B) Transforming growth factor-beta (TGF-B)
(C) Tumor necrosis factor-alpha (TNF-B)
(D) Vascular endothelial growth factor (VEGF)

A

The correct response is Option B.

Transforming growth factor-beta (TGF-B) has been shown to stimulate fibroblasts to produce collagen. This factor is one of several signaling molecules and is produced by mesenchymal cells. The epineurial scarring that occurs following injury to peripheral nerves leads to deposition of type I collagen within fibroblasts, subsequently resulting in inhibition of axonal regeneration. Studies have shown that antibody blockage of TGF-B is clinically beneficial in facilitating optimal axonal regeneration after injury.

Platelet-derived growth factor (PDGF), tumor necrosis factor alpha (TNF-B), and vascular endothelial growth factor (VEGF) have not been shown to affect fibroblast deposition of collagen. Instead, these factors produce a variety of end cellular responses.

109
Q

A 26-year-old man sustains circumferential abrasions and lacerations to the right arm in a roll-over motor vehicle collision. On examination, the arm is covered in dirt and debris. In addition to irrigation of the wound site, which of the following is the most appropriate initial management?

(A) Immediate closure
(B) Operative closure
(C) Immediate split-thickness skin grafting
(D) Daily whirlpool hydrotherapy
(E) Mechanical debridement

A

The correct response is Option E.

The most appropriate management of this patient is irrigation and mechanical debridement of the wound site. Patients with soft-tissue lacerations covered with debris often have foreign particles embedded within the dermis or subcutaneous tissue. If this material is not removed promptly, a traumatic tattoo will ultimately develop; treatment of this complication is difficult and frequently unsuccessful. Therefore, mechanical devices, such as scrub brushes or pulse irrigation devices, should be used with physical retrieval to ensure that all debris is removed.

Coverage of the extremity with any type of dressing will not address the embedded particulate matter. Hydrotherapy may be useful in removing surface debris but not subcutaneous debris.

110
Q

Administration of which of the following vitamins to a surgical wound has been shown to reverse the adverse effects associated with corticosteroid use?

(A) Vitamin A
(B) Vitamin B6
(C) Vitamin B12
(D) Vitamin C
(E) Vitamin E

A

The correct response is Option A.

The negative effects on wound healing resulting from corticosteroid use occur secondary to an arrested inflammatory phase. Corticosteroids inhibit wound macrophages and disrupt the mechanisms of fibrogenesis, endogenesis, and wound contraction. Vitamin A restores the monocytic inflammation process that is inhibited by the use of corticosteroids, although its mechanism of action is not fully understood. A dose of 25,000 IU of vitamin A daily for three to five days is recommended.

111
Q

Which of the following substances has been shown to occur in higher levels in keloids and red hypertrophic scars than in pink or white hypertrophic scars?

(A) Adenosine triphosphate
(B) Creatine kinase
(C) Fibronectin
(D) Guanosine triphosphate

A

The correct response is Option A.

When compared with more mature pink and white scars, keloids and red hypertrophic scars have been shown to have higher levels of adenosine triphosphate. In addition, greater quantities of fibroblasts have also been found in keloid scars when compared with more mature scars. Both keloids and hypertrophic scars actively synthesize collagen fibers and have been shown to have increased activity of glycolytic enzymes in vivo.

Creatine kinase, fibronectin, and guanosine triphosphate have not been shown to be present at higher levels in keloids or red hypertrophic scars.

112
Q

In patients with vitamin C deficiency, which of the following physiologic findings is most likely to be decreased?

(A) Amino acid hydroxylation
(B) Fibronectin production
(C) Helical integrity
(D) Monocyte activation
(E) Prothrombin production

A

The correct response is Option A.

In a patient who has a vitamin C (ascorbic acid) deficiency, hydroxylation of amino acids such as lysine and proline is likely to be decreased. Hydroxylysine and hydroxyproline are the primary components of collagen, with hydroxylysine being responsible for the formation of covalent crosslinks. Vitamin C deficiency, otherwise known as scurvy, prevents collagen cross-linking and the maturation phase of wound healing, resulting in collagen breakdown. Both humans and guinea pigs are unable to manufacture vitamin C naturally.

Decreases in fibronectin production and monocyte activation are the hallmarks of vitamin A deficiency. Prothrombin production is inhibited in patients with vitamin K deficiency. Helical integrity is not affected by vitamin deficiencies.

113
Q

For each phase of wound healing, select the most closely associated cell type (A-E).

(A) Basal epithelial cell
(B) Fibroblast
(C) Platelet
(D) Macrophage
(E) Smooth muscle cell

1) Inflammation phase
2) Proliferative phase

A

The correct response for Item 1 is Option D and for Item 2 is Option B.

Inflammation is the initial phase of wound healing; polymorphonuclear leukocytes (PMNs) and macrophages are primarily involved in this process. This interval typically lasts from four to six days; during this time, clots, foreign bodies, and bacteria are removed, and the wound surface closes. The proliferative phase, also known as the collagen and regenerative phase, is characterized by collagen production and increased strength within the wound. Fibroblasts are primarily responsible for the collagen production. This process begins approximately seven days after injury; its duration is approximately five weeks. Maturation, or remodeling, is the final phase and can last for more than two years. Maturation and cross linking of collagen occur during this phase.

114
Q

Which of the following is the most likely result following the intralesional injection of corticosteroids for treatment of keloids?

(A) Absence of adverse effects on the surrounding tissues
(B) Decreased risk for malignant degeneration
(C) Decreased risk for recurrence
(D) Lack of effectiveness on the connective tissue composition of the keloid
(E) Symptomatic relief of itching and burning

A

The correct response is Option E.

Intralesional corticosteroid injections are among several therapies used for treatment of keloids. Other therapeutic modalities include application of occlusive silicone dressings, use of compressive pressure earrings or dressings, interferon therapy, radiation therapy, cryosurgery, and laser or surgical excision. None of these treatments have been shown to be totally effective; however, corticosteroids have been shown to relieve the itching and burning symptoms associated with the keloids, as well as to decrease the collagen content of the keloids and subsequently decrease their size. Excision performed concomitantly with injection of corticosteroids will reduce the rate of recurrence to 30% to 50%. Low-dose radiation therapy administered postoperatively is associated with a similarly reduced rate of recurrence.

115
Q

Which of the following is the most likely result following the intralesional injection of corticosteroids for treatment of keloids?

(A) Absence of adverse effects on the surrounding tissues
(B) Decreased risk for malignant degeneration
(C) Decreased risk for recurrence
(D) Lack of effectiveness on the connective tissue composition of the keloid
(E) Symptomatic relief of itching and burning

A

The correct response is Option E.

Intralesional corticosteroid injections are among several therapies used for treatment of keloids. Other therapeutic modalities include application of occlusive silicone dressings, use of compressive pressure earrings or dressings, interferon therapy, radiation therapy, cryosurgery, and laser or surgical excision. None of these treatments have been shown to be totally effective; however, corticosteroids have been shown to relieve the itching and burning symptoms associated with the keloids, as well as to decrease the collagen content of the keloids and subsequently decrease their size. Excision performed concomitantly with injection of corticosteroids will reduce the rate of recurrence to 30% to 50%. Low-dose radiation therapy administered postoperatively is associated with a similarly reduced rate of recurrence.

116
Q

Which of the following is the most likely mechanism of action of silicone sheeting/silicone gel pads in enhancing scar maturation?

A) Decreasing wound tension

B) Deregulating cellular integrins

C) Enhancing epidermal contact inhibition

D) Increasing the static electronegative field

E) Maintaning regulated wound temperature

A

The correct response is Option D.

Silicone sheeting and silicone gel pads are used to treat hypertrophic or immature scars and keloids. Although their exact mechanism of action is unknown, some surgeons postulate that their positive effect is associated with the generation of an increased static electronegative field by the silicone. This mechanism of action results in favorable wound effects. Other theories propose that the wound-healing mechanism is related to the decreased oxygenation, sustained pressure, or hydrating effects of silicone oil resulting from the use of these products.

Silicone sheeting and gel pads have not been shown to decrease wound tension, affect epidermal contact inhibition, or regulate intracellular integrins or wound temperature.

117
Q

Which of the following impairs the process of epithelialization during wound healing?

(A) Basic fibroblast growth factor
(B) Epidermal growth factor
(C) Isotretinoin
(D) Keratinocyte growth factor
(E) Tretinoin

A

The correct response is Option C.

Isotretinoin is the only agent of those listed that impairs epithelialization instead of promoting it. Isotretinoin (13-cis retinoic acid, or Accutane) is a retinoid, one of a family of vitamin A-related agents. Because of its antikeratinization effect, which results in thinning of the stratum corneum and decreased activity of skin appendages such as sebaceous glands, as well as its effect on wound epithelialization, it is used in the treatment of cystic acne. In addition, patients who have been taking isotretinoin experience delayed or poor wound healing following chemical peeling or laser skin resurfacing because of the effect on wound epithelialization. Therefore, it is recommended that isotretinoin be discontinued a minimum of one year before chemical peeling or laser peeling is performed.

Basic fibroblast growth factor is a polypeptide and a member of the family of fibroblast growth factors (FGF). This agent stimulates important aspects of wound healing, including angiogenesis, collagen and collagen matrix syntheses, wound contraction, and epithelialization.

Epidermal growth factor is a polypeptide FGF that affects endothelial cells, fibroblasts, and smooth muscle cells. Because epithelial cells have been shown to have the greatest number of receptors for epidermal growth factor, the primary effect of epidermal growth factor is believed to be promotion of epithelialization.

Keratinocyte growth factor is produced by fibroblasts and also primarily affects epithelialization; only epithelial cells have keratinocyte growth factor receptors. Delayed wound healing has been reported in transgenic animals that lack this signaling receptor.

Although tretinoin is also classified as a retinoid, its effects are far different than isotretinoin. Tretinoin (all-trans-retinoic acid, Retin-A) promotes epithelialization by stimulating mitotic activity and decreasing the turnover of follicular epithelial cells. As a result, tretinoin is often used as a pretreatment in patients undergoing chemical peeling and laser skin resurfacing to accelerate wound healing. Other conditions for which tretinoin has proved beneficial include skin aging, acne vulgaris, and dysplastic nevus syndrome, as well premalignant and malignant tumors such as actinic keratosis, carcinoma in situ, and superficial basal cell carcinoma.

118
Q

Which of the following is the predominant type of collagen found in basement membrane?

(A) Type I
(B) Type II
(C) Type III
(D) Type IV
(E) Type V

A

The correct response is Option D.

Type IV collagen is the predominant collagen in basement membrane. In contrast, type I collagen is most often found in normal, mature skin, as well as in tendon and bone. Type II collagen is present in hyaline cartilage and the tissues of the eye. Type III collagen is located in the papillary dermis, arteries, intestinal walls, and uterus. In addition, hypertrophic and immature scars can contain as much as 30% type III collagen. Type V collagen is also found within the basement membrane in lesser amounts than type IV collagen.

119
Q

Which of the following sites is most susceptible to the development of a keloid following injury?

(A) Eyelid
(B) Genitalia
(C) Upper arm
(D) Palm
(E) Sole

A

Which of the following sites is most susceptible to the development of a keloid following injury?

(A) Eyelid
(B) Genitalia
(C) Upper arm
(D) Palm
(E) Sole

120
Q

In patients who exhibit allergic sensitivity to bovine collagen, which of the following types of immunologic response is most common?

(A) IgA antibodies
(B) IgD antibodies
(C) IgE antibodies
(D) IgG antibodies
(E) IgM antibodies

A

The correct response is Option D.

Anti-bovine collagen (Zyderm) antibodies are classified as IgG antibodies. Zyderm is a purified form of bovine collagen that consists of 95% type I collagen with 5% type II collagen. It is available in two concentrations, 35 mg/mL and 65 mg/mL, as well as in a glutaraldehyde cross-linked form known as Zyplast, which in theory degrades more slowly. Enzymatic processing is used to remove the nonhelical portion of the collagen molecule, thus reducing most of its associated antigenicity.

These various forms of injectable collagen are used for correction of depressed scars, shallow or soft acne scars, and fine facial rhytids associated with aging. Ice pick acne scars cannot be treated with collagen injections. In patients undergoing treatment, the collagen is injected intradermally in excess amounts, which are necessary to compensate for absorption of the saline component of the solution. Some of the injected collagen is lost over the next six to nine months as collagen breakdown occurs.

Because approximately 3% of all treated patients will have an allergic reaction to injectable bovine collagen, skin testing should be performed prior to any treatment. Following intradermal injection of a test dose into the volar forearm, the patient should be assessed 72 hours after injection and again at four weeks after injection, as any adverse changes noted at the test site may indicate an allergic reaction. This is defined as the onset of erythema, induration, tenderness, or swelling to any degree, with or without pruritus, that appears more than 24 hours after injection and/or persists longer than six hours.

Approximately 66% of those patients who are allergic to injectable collagen will have a positive reaction within 72 hours, while 33% will develop positive findings within four weeks. An additional 1% will have negative findings on skin testing but will subsequently develop an allergic reaction following injection. One study of those patients who had negative skin tests and subsequent allergic reactions showed that 56% developed a reaction following the first treatment, while 28% experienced the reaction after two treatments. All of the patients who exhibited allergic sensitivity developed IgG antibodies against bovine collagen. In contrast, 50% developed IgA antibodies; IgD, IgE, and IgM antibodies were not identified.

121
Q

In normal wound healing, collagen synthesis and collagen breakdown typically reach a state of equilibrium approximately how many days after injury?

(A) 7
(B) 14
(C) 21
(D) 60
(E) 90

A

The correct response is Option C.

In normal wound healing, collagen synthesis and collagen breakdown typically reach a state of equilibrium approximately 21 days after initial injury. Collagen synthesis depends primarily on production of procollagen by fibroblasts. This procollagen is inserted into secretory vessels that move toward the cell surface. It then is cleaved into collagen at the level of the cell membrane, and the collagen is then released into the wound. Macrophages help to regulate collagen synthesis by producing growth factors that stimulate fibroblast proliferation and subsequent collagen production.

In collagen degradation, fibroblasts, granulocytes, macrophages, and other cells produce specific matrix metalloproteinases (MMP) at the wound site. The MMP family of zinc-dependent endopeptidases includes collagenase, gelatinase, and stromelysin. Several members of the MMP family have been linked to chronic wounds; these substances, such as MMP-2 and MMP-9, have been shown to be absent in acute wounds. A higher turnover of extracellular matrix is thought to contribute to the delayed healing or nonhealing seen in association with chronic wounds. Transforming growth factor-beta can be used to combat this; it has been shown to decrease MMP activity and increase the activity of MMP inhibitors.